You are on page 1of 113

The Complete (but not obsessive) Hematopathology Guide

page 2
www.pathologystudent.com







About t hi s gui de
If youre reading this introduction, it means you are probably either a) covering hematopathology in
your pathology class right now, or b) studying for boards. Either way, youve come to the right
study guide! Inside, youll find a comprehensive (but not oppressive) review of both benign and
malignant hematopathology, neatly summarized and nicely illustrated. Whether you have a read-
the-text-straight-through kind of mind, or a looking-at-pictures mind, or a question-working mind,
youll find it easy to work your way through this guide.

Ext ra hel p
If you are stuck, or frustrated, or if something just doesnt make sense, feel free to email me at
pathology@pathologystudent.com. Ill do my best to get you unstuck and back on track.














2011 Pat hol ogy St udent

The Complete (but not obsessive) Hematopathology Guide
page 3
www.pathologystudent.com




Table of Contents


1. I nt roduct i on p. 5
2. Anemi a p. 12
3. Beni gn l eukocyt oses. . p. 39
4. Leukemi a p. 46
Acute myeloid leukemia.. p. 48
Myelodysplastic syndromes p. 59
Acute lymphoblastic leukemia p. 60
Chronic myeloproliferative disorders.. p. 65
Chronic lymphoproliferative disorders p. 72
5. Myel oma. . p. 78
6. Lymph node di sorders. p. 80
Benign lymph node disorders. p. 80
Non-Hodgkin lymphoma. p. 83
Hodgkin disease.. p. 93
7. Ref erence sect i on. p. 96
8. St udy quest i ons. p. 102

The Complete (but not obsessive) Hematopathology Guide
page 4
www.pathologystudent.com



List of Diseases


























Anemi a
Iron-deficiency anemia
Megaloblastic anemia
Hemolytic anemias
Hereditary spherocytosis
G6PD deficiency
Sickle cell anemia
Thalassemia
Warm autoimmune hemolytic anemia
Cold autoimmune hemolytic anemia
Microangiopathic hemolytic anemia
Anemia of chronic disease
Anemia of chronic renal disease
Anemia of chronic liver disease
Aplastic anemia
Beni gn l eukocyt oses
Benign neutrophilia
Benign lymphocytosis
Other leukocytoses
Leukemi a
Acute myeloid leukemia
AML with genetic abnormalities
AML with FLT3 mutation
AML with multilineage dysplasia
AML, therapy-related
AML, not otherwise classified
Myelodysplastic syndromes
Acute lymphoblastic leukemia
T-cell ALL
B-cell precursor ALL
Burkitt leukemia
Chronic myeloproliferative disorders
Chronic myeloid leukemia
Chronic myelofibrosis
Polycythemia vera
Essential thrombocythemia
Chronic lymphoproliferative disorders
Chronic lymphocytic leukemia
Hairy cell leukemia
Prolymphocytic leukemia
Large granular lymphocyte leukemia

Myel oma
Lymph node di sorders
Benign lymph node disorders
Non-Hodgkin lymphoma
Small lymphocytic lymphoma
Marginal zone lymphoma
Mantle cell lymphoma
Follicular lymphoma
Mycosis fungoides/Szary syndrome
Diffuse large B-cell lymphoma
Lymphoblastic lymphoma
Burkitt lymphoma
Adult T-cell leukemia/lymphoma
Hodgkin disease
Nodular lymphocyte predominance
Nodular sclerosis
Mixed cellularity
Lymphocyte rich
Lymphocyte depletion


The Complete (but not obsessive) Hematopathology Guide
page 5
www.pathologystudent.com
One. I ntroducti on

Clinical stuff
Patients with hematologic disorders have lots of widely-varying signs and symptoms! Heres a short list
of things to look for in the history and physical. More details will be discussed as we go through
individual disorders.















Physi cal exam

Exami ne Look f or Mi ght mean
Sclerae Jaundice Hemolysis
Petechiae Bleeding disorder
Tongue Beefiness Megaloblastic anemia
Smooth surface Iron-deficiency anemia
Skin Pallor Anemia
Jaundice Hemolysis
Petechiae Bleeding disorder
Nails Spoon shape Iron-deficiency anemia
Heart Tachycardia Severe anemia
Nodes Enlargement Infection or lymphoma
Bones Tenderness Marrow expansion
Multiple myeloma
Spleen, liver Enlargement Leukemia, lymphoma
Nerves Decreased Megaloblastic anemia
vibratory sense
Hi st ory

Ask about Mi ght mean
Infections (repeated or unusual) Leukemia, lymphoma
Bleeding (epistaxis, bleeding gums, Thrombocytopenia, leukemia,
menorrhagia, hemarthrosis) bleeding disorder
Dyspnea, chest pain Anemia
Pica Iron-deficiency anemia
Abdominal fullness/early satiety Splenomegaly
Alcohol use (excessive) Megaloblastic anemia
Headache, neurologic deficits Leukostasis, thrombosis
Pruritis Polycythemia
Prior malignancies, chemotherapy Secondary malignancies
History of thrombosis Factor V Leiden
Family history of bleeding disorder Hemophilia, von Willebrand
disease
Family history of anemia Hemoglobinopathy, thalassemia

The Complete (but not obsessive) Hematopathology Guide
page 6
www.pathologystudent.com

The Complete Blood Count (CBC)
The CBC is comprised of a bunch of different indices. You get all of these on every report, whether
you ask for them specifically or not (thats just the way the machine does it!). Some of these
indices are really useful (like the hemoglobin, MCV and RDW), and some of them are rarely if ever
used (like the mean platelet volume). You should know what each one measures, and be able to
recognize the normal range.

Red bl ood cel l count ( RBC)
Total number of red blood cells in blood
Normal ranges: male 4.5-6.0 x 10
12
/L, female 3.8-5.2 x 10
12
/L

Hemogl obi n ( Hgb)
Concentration of hemoglobin in blood
Normal ranges: male 13-18 g/dL; female 12-16 g/dL
Hgb below normal = anemia

Hemat ocri t ( Hct )
Volume of packed red blood cells.
In the old days, was performed by spinning a tube of blood and estimating the amount of
total blood volume taken up by the red cells (not such a great method because if the cells
are of unusual shape, they may not pack as well as normal red cells, producing an
artificially elevated Hct)
Now calculated by machine (MCV x RBC)
Normal ranges: male 40-52%, female 35-47%

Mean red bl ood cel l vol ume ( MCV)
Average size of red blood cells
Normal range: 80-100 fL (1 fL = 10
-15
L)
Differentiates between microcytic (MCV < 80), normocytic (MCV 80-100) and macrocytic
(MCV > 100) anemias

The most useful red cell indices
are the hemoglobin, MCV and RDW.

The Complete (but not obsessive) Hematopathology Guide
page 7
www.pathologystudent.com

Mean cel l hemogl obi n ( MCH)
Weight of Hgb in the average red blood cell
Normal range: 26-34 pg (1 pg = 10
-12
g)
Not a frequently used parameter
Mean cel l Hgb concent rat i on ( MCHC)
Concentration of Hgb in the average red blood cell
Normal range: 32-36 g/dL
Calculated by machine (Hgb/Hct)
Used to differentiate between hypochromic (MCHC < 32) and normochromic (MCHC 32-
36) anemias
There is no such thing as a hyperchromic red cell.
You can see this nicely on a blood smear: normochromic cells have a zone of central
pallor (that white dot in the middle of the cell) that is no more than 1/3 the diameter of the
red cell. Hypochromic red cells have just a thin rim of hemoglobin.
Red cel l di st ri but i on wi dt h ( RDW)
Standard deviation of the MCV
Tells you how much the red blood cells differ from each other in size. If they are all pretty
similar in size, the RDW is low. If some are little and some are big, the RDW is high.
Normal range = 12-13.5%
Used to differentiate between anemias with minimal anisocytosis (difference in cell size)
(RDW 12-13.5%) and those with increased anisocytosis (RDW > 13.5%).
You can see this on a blood smear: when anisocytosis is increased, youll see a range of
cell sizes some are smaller, some are bigger.

Whi t e bl ood cel l count ( WBC)
Total number of leukocytes in blood
Normal ranges: adult: 4.5-11 x 10
9
/L, newborn: 9 -30, child over 1: 5.0-17.0
A high WBC is seen in many conditions. Some are benign, such as infection and
inflammation (see the Benign Leukocytosis section of this book). Others are malignant,
such as leukemia (see the Leukemia section of this book).



The Complete (but not obsessive) Hematopathology Guide
page 8
www.pathologystudent.com



Di f f erent i al ( di f f )
Amounts of each white blood cell type in blood
Note: dont just look at the percentages. Look at the absolute values too, particularly if the
WBC is not exactly normal. You can get thrown off if you just look at the percentages. For
example, if your patient has a WBC of 1, and the % neutrophils is 50, you might think the
neutrophil count is normal (but it isnt! That would be an absolute neutrophil count of .5,
which is definitely low).
Normal ranges:

Percentage of WBC Absolute (x 10
9
/L)
Neutrophils 45-75 2-8
Lympocytes 20-50 1-4
Monocytes 1-8 0.1-0.8
Eosinophils 0-6 0-0.5
Basophils 0-1 0-0.3


Pl at el et count ( Pl t )
Total number of platelets in blood
Normal range = 150-450 x 10
9
/L
Causes of a low platelet count are numerous and include splenomegaly, idiopathic
thrombocytopenic purpura, disseminated intravascular coagulation, and bone marrow
failure. Causes of a high platelet count are also numerous, and include reactive
thrombocytosis (as seen in iron-deficiency anemia) and essential thrombocythemia.

MPV ( mean pl at el et vol ume)
Average size of platelets
Normal range depends on the platelet count! (Normally, if the platelet count falls, the body
compensates a little by trying to make bigger platelets.)
Not used all that often.


The Complete (but not obsessive) Hematopathology Guide
page 9
www.pathologystudent.com
The Blood Smear
There are three main things you need to look at when youre faced with a blood smear: the red
cells, the white cells, and the platelets. It helps if you have a plan that you follow every time, kind of
like radiologists do when they look at imaging studies. That way youre not tempted to just start
looking at the exciting stuff (like weird looking neutrophils, for example) and forget about all the
other stuff. So heres your plan.

Look at the red bl ood cel l s
1. Est i mat e number.
Just eyeball it; make sure there arent a lot of holes between the cells).
2. Look f or vari at i on i n si ze ( ani socyt osi s) .
Oval macrocytes (B
12
/folate deficiency)
Microcytes (iron deficiency anemia, thalassemia)
The size range can often help you narrow down which type of anemia is present
(for example, in iron-deficiency anemia, there is usually a big range of sizes)
3. Look f or vari at i on i n shape ( poi ki l ocyt osi s) .
Schistocytes (microangiopathic hemolytic anemia)
Spherocytes (hemolytic anemia, hereditary spherocytosis)
Teardrop cells or dacryocytes (myelofibrosis or myelophthisic processes)
Target cells or codocytes (hemoglobinopathies, thalassemias, liver disease)
Sickle cells (sickle cell anemia)
Echinocytes and acanthocytes (liver disease)
4. Est i mat e t he average amount of hemogl obi n i n each cel l ( chromasi a) .
Normochromic (zone of central pallor comprises ! 1/3 of the cell diameter)
Hypochromic (zone of central pallor comprises >1/3 of the cell diameter)
5. Est i mat e number of ret i cul ocyt es ( l ook f or pol ychromat ophi l i c cel l s) .
Normal: one or two polychromatophilic cells per field.
The lower the Hgb, the higher the reticulocyte count should be.
6. Look f or anyt hi ng el se wei rd.
Nucleated red blood cells
Inclusions (Howell-Jolly bodies, Pappenheimer bodies, bugs)
Things to make you look smart
Q. What s t he di f f erence
between a pol ychromatophi l i c
cel l and a reti cul ocyte?
A. Polychromatophilic cell and
reticulocyte are two names for the
same thing immature red cells that
still contain a little ribosomal RNA.
When you do a normal Wright-Giemsa
stain, the cells look big and slightly
basophilic, and they are called
polychromatophilic cells. When you
do a supravital stain on a blood
smear, the RNA stains blue, and the
cells are called reticulocytes. This is
one of those fine points that, when
casually mentioned on rounds your
third or fourth year, will make you look
crazy smart.
When looking at blood smears,
every time.
it helps to follow the same plan

The Complete (but not obsessive) Hematopathology Guide
page 10
www.pathologystudent.com

Look at the pl atel ets
1. Estimate number (you should see between 7 and 21 platelets per high power field).
2. Check morphology (size, granulation).

Look at the whi te bl ood cel l s
1. Estimate number (you should see no more than a few white cells per high power field).
2. Check morphology (make sure the neutrophils and lymphocytes look normal, and that there
arent any FLCs (Funny-Looking Cells, like blasts, for example, or circulating carcinoma cells).
3. Do a differential count (you should count at least 200, and preferably 500, cells).

The Bone Marrow Biopsy
When a malignant hematopoietic disorder is suspected, a bone marrow biopsy is usually
performed. The patient lies on his or her stomach, and following a little lidocaine to the skin and
periosteal bone, a core sample of bone is obtained, followed by an aspiration sample of liquid
marrow.
The core biopsy (also called a trephine biopsy) is evaluated for cellularity (your marrow becomes
more fatty as you age, just like every other body part), for the myeloid-to-erythroid ratio (which
should be around 2:1), and for anything else funny looking (like lymphoid aggregates or metastatic
tumor). The aspirate sample is evaluated like a blood smear; a differential count is performed
(usually you count a lot more cells, like, say, 1000) and the cells are checked to make sure they
look normal.
Bone marrow t rephi ne bi opsy sect i on
Bone marrow aspi rat e smear,
l ow ( t op) and hi gh ( bot t om) power

The Complete (but not obsessive) Hematopathology Guide
page 11
www.pathologystudent.com
Speci al Studi es
Depending on what disorder youre looking for, you may wish to order one or more special
studies, which are usually performed on the bone marrow aspirate specimen. There are lots of
possibilities, but here are the most common ones.

I mmunophenot ypi ng
Immunophenotyping, or flow cytometry, is done on a big machine (called, not surprisingly, a flow
cytometer). Its done to look for markers on the surface of cells little molecules that give clues
as to what type of cell youre dealing with. These are given CD number designations. Over on the
right are some common markers.

Cyt ogenet i cs
Cytogenetic assays are performed by taking a sample of cells, growing them up in culture, and
photographing them right at that point in metaphase when all the chromosomes are all lined up on
the mitotic spindle so you can see them. Then you cut out the little chromosomes (on the
computer), pair them up (you have to have a lot of training to do this!), and line them up in whats
called a karyotype. This is cool because it gives you a good overall look at all of the chromosomes.
If there are changes like deletions or translocations, you can often see those changes in a
karyotype. On the right are some common chromosomal abnormalities.
The nice thing about cytogenetics is that you dont really need to know what youre looking for,
because youre looking at all the chromosomes (this is a great benefit in following patients with
hematologic malignancies, because sometimes they develop chromosomal abnormalities that you
werent even expecting). On the other hand, its not the most sensitive test in the world. So if you
dont happen to get a malignant cell in your little growing group of culture cells, then you wont get
to look at the chromosomes in the malignant population. Thats where the next set of tests comes
in.

Mol ecul ar st udi es
The whole point of molecular studies is to look for a specific DNA sequence, like the particular
sequence of DNA encompassing the bcr-abl translocation in CML. These tests are cool because
they are generally really sensitive. Polymerase chain reaction (or PCR), in particular, is super
sensitive. You only need one copy of a particular DNA sequence in your sample PCR will amplify
it over and over and over so you have a bazillion copies and you can easily detect it! This is of
great benefit in certain settings, like when youre following someone to make sure that all the
disease is gone. If you cant even detect the disease using PCR, well, then its really, really gone.
Marker Expressed by:
CD2 all T cells
CD3 all peripheral (post-
thymic, mature) T cells
CD5 all T cells
CD10 developing B cells
CD19 developing and mature
B cells
CD20 developing and mature
B cells
CD45 all leukocytes
CD13 granulocytes and
monocytes
CD14 monocytes
CD15 granulocytes
CD33 myeloid precursors and
monocytes
CD41 megakaryocytes and
platelets
CD61 megakaryocytes and
platelets
Di sorder Abnormal i ty
CML t(9;22)
CLL trisomy 12
AML-M2 t(8.21)
AML-M3 t(15;17)
AML-M4 inv(16)
AML-M4 and M5 11q23
T-cell ALL t(11;14)
B-cell precursor ALL t(9;22)
B-cell ALL t(8;14)

The Complete (but not obsessive) Hematopathology Guide
page 12
www.pathologystudent.com


Two. Anemi a

Anemia (from an-, without, and -emia, blood) is a reduction below normal in hemoglobin or red
blood cell number. There are lots of different kinds of anemia, but before we leap into that
discussion, lets take a look at some ways of thinking about anemia. Its always a good idea to
have an underlying understanding, or plan, when youre approaching a sh*tload of diseases. Who
can memorize 42 separate things? Well, okay, you can, or you wouldnt be here. But it makes it
easier if you can group those 42 things together into, say, 3 or 4 bigger categories.

Causes
One way to group the anemias is by their cause. If you think about it, there are basically only three
ways you can get anemic. All the anemias well talk about fit into these three mechanisms:
1. Lose bl ood (for example, as a result of major trauma)
2. Dest roy t oo much bl ood
Extracorpuscular hemolytic disease (e.g., autoimmune disorders, trauma to red blood cells)
Intracorpuscular hemolytic disease (e.g., hereditary membrane or globin problems)
3. Make t oo l i t t l e bl ood
Bad diet (not enough iron, B
12
, or folate)
Decreased number of erythroblasts (as in aplastic anemia)
Bone marrow full of other stuff besides hematopoietic precursors (e.g., tumor)
Chronic disease (e.g., renal disease, inflammatory diseases)






The Complete (but not obsessive) Hematopathology Guide
page 13
www.pathologystudent.com
Morphol ogi c groups
Another way to think about the anemias is in terms of the way they look on a blood smear. Which
actually makes some sense, because when a patient comes in, he or she doesnt say, I have an
anemia due to lack of folate what happens is that you suspect anemia, and then look at their
blood smear to try to figure it out. It doesnt really matter which little grouping scheme you choose,
just pick one that makes sense to you. In this study guide, the anemias are loosely organized by
the morphologic group scheme.
1. Anemi as wi t h abnormal l y-si zed red cel l s
Red cells too small (microcytic) (iron-deficiency anemia, thalassemia)
Red cells too big (macrocytic) (megaloblastic anemia)
2. Anemi as wi t h abnormal l y-shaped red cel l s
Round red cells (spherocytes) (hereditary spherocytosis, autoimmune hemolytic anemia)
Pointy red cells (sickle cell anemia, G6PD deficiency, microangiopathic hemolytic anemia)
Target-shaped red cells (hemoglobinopathies, thalassemias)
3. Anemi as wi t h normal -l ooki ng red cel l s
Increased reticulocytes (massive hemorrhage that occurred over 3 days ago)
No increase in reticulocytes (recent massive hemorrhage, anemia of chronic disease,
aplastic anemia)

Si gns and symptoms
One final note before we get on to the specific anemias. Patients with anemia can present in
different ways, depending on what kind of anemia they have and how severe it is. The general
signs and symptoms of anemia relate to the underlying lack of oxygen-carrying capacity: fatigue,
weakness, dizziness, tachycardia, pallor of skin and mucous membranes.
In addition to the general symptoms of anemia, some specific findings may be present. If the
anemia is hemolytic, the patient may be jaundiced. Patients with iron-deficiency anemia may show
spoon-shaped nails (koilonychia), a smooth tongue, or pica (a craving to eat dirt and other non-
food items). And patients with megaloblastic anemia may develop a big, beefy tongue, or loss of
vibratory sensation or proprioception.
Its important to remember, however, that if an anemia is fairly mild, symptoms may not be present
at all. Also, if the anemia is chronic and slowly-progressive, the cardiovascular system adjusts to
the new diminished level of oxygen, and symptoms will only appear when the anemia becomes
quite severe.

The Complete (but not obsessive) Hematopathology Guide
page 14
www.pathologystudent.com




Iron-Deficiency Anemia

Iron-deficiency anemia is a common type of anemia. One of the most common causes is blood
(and hence iron) loss so its important to work up any patient to find a source of blood loss, if it
exists. Occasionally, colon cancer presents with iron-deficiency anemia due to chronic loss of
small amounts of blood in the stool.

I ron Facts
Absorpt i on
Absorption of iron occurs in duodenum/proximal jejunum. In the mucosal cell, iron is bound to
either ferritin (for storage) or transferrin (for circulation).
Ci rcul at i on
Iron is bound to transferrin, which carries iron to red blood cell precursors in bone marrow, and to
other organs.
Di st ri but i on
Most of the iron in the body is in hemoglobin; a smaller percentage is in storage forms (ferritin and
hemosiderin), and a very very small amount is bound to transferrin.
Met abol i sm
Most of the circulating iron is taken up by red cell precursors and incorporated into heme (which is
then combined with globin chains to make hemoglobin). The rest of the iron is stored in
macrophages in the marrow, spleen, and liver.
St orage
There are a couple ways iron is stored: in ferritin (a labile iron storage form quick in, quick out)
and in hemosiderin (a stable storage form that contains ferritin and cell debris).


The Complete (but not obsessive) Hematopathology Guide
page 15
www.pathologystudent.com

Causes of I ron Def i ci ency
Decreased i ron i nt ake
Dietary deficiency is rarely the sole cause of iron deficiency anemia (unless you are eating a really
crappy diet). More likely is decreased absorption, for example, in patients who have had gastric
surgery.
I ncreased i ron l oss
Heres the scary potential cause of iron deficiency: GI bleeding (e.g., from gastric ulcer, colon
cancer). This is why you never just treat an iron-deficient person with iron without figuring out the
cause of the iron deficiency! You might miss something really important that way. Other causes of
bleeding that are readily identifiable and usually less scary include excessive menstrual flow
(menorrhagia) and acute blood loss (e.g., massive trauma, childbirth).
I ncreased i ron requi rement
Women who are pregnant need more iron; if their intake doesnt increase accordingly, a deficiency
can arise.
I t al l boi l s down to thi s:
1. IDA in premenopausal women: First thing to consider is menorrhagia.
2. IDA in men and postmenopausal women: First thing to consider is GI blood loss.

Cl i ni cal Features
Sympt oms
Typical symptoms of iron-deficiency anemia, like any type of anemia, are related to the lack of
ability to carry oxygen around! So patients have fatigue, palpitations, dizziness, and
breathlessness. Note, though, that if the anemia is mild (say, with a hemoglobin over 8), or if its
long-standing (say, from a slow GI bleed), the patient might have few or no symptoms at all!
Si gns
Patients with anemia in general (whatever the cause) may show pale skin and mucous
membranes. Patients with anemia due to iron deficiency may have some unique symptoms, such
as thinned, flattened, spoon-shaped nails, or a smooth, shiny tongue. A really strange symptom of
iron deficiency that occurs in some patients is pica. Hippocrates described pica as "a craving to
eat the earth" associated with "corruption of the blood." Good description. For whatever reason,
some patients with severe iron deficiency get cravings to eat weird stuff, like dirt, ice, cardboard,
and Windex. Of course, there are other causes of pica too, some of them psychiatric in nature.
Bottom line:
IDA in men and postmenopausal women:
rule out GI blood loss.
Things to make you look smart
Q. Why does t he pat i ent ' s
hemogl obi n appear normal
i mmedi at el y af t er a bi g bl ood
l oss?
A. Because the patient has lost not
only red cells, but plasma. So the
blood that's left in the body has the
same concentration of hemoglobin as
it did before the blood loss. After a
while, plasma volume is restored
(either artificially or by the body itself),
and the hemoglobin (now diluted) is
decreased.

The Complete (but not obsessive) Hematopathology Guide
page 16
www.pathologystudent.com


Morphol ogy
Bl ood
Hypochromic, microcytic anemia
Increased anisocytosis (some little cells, some bigger cells)
Increased poikilocytosis (elliptocytes are present, for some reason)
Decreased reticulocyte number (because theres not enough iron around!)
Platelet count usually increased for some reason
Bone marrow
Erythroid hypoplasia
Dyserythropoiesis (funny looking red precursors)

I ron studi es
" serum iron
# TIBC (total iron binding capacity)
" ferritin (But remember: ferritin is an acute phase reactant, which means it goes up in certain
settings, like inflammation. So a normal ferritin (or even an increased ferritin) doesnt rule out iron
deficiency!)

Treatment
Figure out why patient is iron deficient (don't just treat the anemia, or you might miss something
really important). Then give iron (orally).



I ron-def i ci ency anemi a
Iron-deficiency anemia is a
microcytic, hypochromic anemia with
oval macrocytes
increased anisocytosis and poikilocytosis.

The Complete (but not obsessive) Hematopathology Guide
page 17
www.pathologystudent.com



Megaloblastic Anemia

Patients with megaloblastic anemia are unable to make DNA at a normal rate, leading to big,
weird-looking cells with immature nuclei but mature cytoplasm. The most common cause is B
12

and/or folate deficiency.

Pathogenesi s
Ret arded DNA synt hesi s (causing cells to divide more slowly), but unimpaired RNA synthesis
(allowing cytoplasm to mature at normal speed) leads to big cells with immature nuclei but mature
cytoplasm (nuclear/cytoplasmic asynchrony).
Vi t ami n B
12
and/or f ol at e def i ci ency is most common cause of retarded DNA synthesis.
You need both B
12
and folate to make DNA:

Methyl FH
4


B
12


FH
4





Methylene FH
4
FH
2



dUMP dTMP DNA



The Complete (but not obsessive) Hematopathology Guide
page 18
www.pathologystudent.com
Vi tami n B
12

Sources
B
12
is found in meat, dairy products, and fortified foods, like breakfast cereal, but not in plants!
How does B
12
get t o red cel l s?
Ingested B
12
binds to intrinsic factor (secreted by gastric parietal cells). B
12
/IF is absorbed in the
distal ileum. B
12
is transported by transcobalamin to organs and erythroblasts.
What el se do you need B
12
f or?
B
12
is also necessary for conversion of homocysteine to methionine. You need methionine for
myelin maintenance (patients with untreated B
12
deficiency eventually get an irreversible
demyelinating disease of the spinal cord called subacute combined degeneration). Soeven if
you know a patient has a folate deficiency, always check for a concurrent B
12
deficiency!
Causes of B
12
def i ci ency
1. Diet. This is a pretty rare cause. If you stopped eating B
12
completely, it would take a few years
to become anemic.
2. Pernicious anemia. Patients have autoantibodies to their parietal cells (as these are destroyed,
less IF is produced). Once dreaded and lethal (pernicious), now easily treated with B
12
injections.
To see how well a patient absorbs B
12
, you can do a Shilling test.
3. Pancreatic damage. Lack of pancreatic enzymes means you cant liberate B
12
in the stomach,
which means that B
12
cant bind to IF.
4. Ileal disease/resection. Without an ileum, B
12
-IF cant be absorbed.
5. Bugs in the small intestine. Tapeworms or bacterial overgrowth can compete for B
12
.

You need B
12
to convert
homocysteine into methionine,
myelin maintenance.
and you need methionine for
Things to make you look smart
Q. How do you do a Shi l l i ng t est ?
A. 1. Give flushing dose of intramuscular B
12
.
2. Give small oral dose of radioactive B
12
.
Healthy patients will excrete radioactive B
12
in urine.
Patients who cant absorb B
12
via gut will not excrete radioactive B
12
in urine.
3. If urine has low radioactivity, give another oral dose of radioactive B
12
, this time with intrinsic factor.
If patient now excretes radioactive B
12
, patient lacks intrinsic factor.
If patient still doesnt excrete radioactive B
12
, the defect is not in intrinsic factor (something else
is probably wrong with absorption).


The Complete (but not obsessive) Hematopathology Guide
page 19
www.pathologystudent.com



Fol ate
Sources
There are lots of dietary sources of folate like green leafy vegetables (folic means leafy),
legumes, yeast, organ meats, fruit, eggs, and fortified products like breakfast cereals.
How does f ol at e get t o red cel l s?
Folate is absorbed mostly in the jejunum. Its converted to methyl-FH
4
during absorption, and then
transported freely (mostly) to liver, red blood cells.
Causes of f ol at e def i ci ency
1. Diet. Folate stores are relatively small! If you stopped eating folate completely, it would only take
a few months to become anemic.
2. Alcohol abuse. Probably due to malnutrition, poor absorption of folate, and/or inhibition of folate
metabolism.
3. Jejunal disease. For example, sprue, inflammatory bowel disease, jejunal resection.
4. Drugs. Especially chemotherapeutic drugs, many of which are folate antagonists.

Morphol ogy
Bl ood
Macrocytic anemia (MCV >100)
Oval macrocytes.
Hypersegmented neutrophils.
Bone marrow
Megaloblastic erythroblasts (BIG cells with big,
immature nucleus but maturing cytoplasm).
Megaloblastic neutrophils and precursors.
Giant metamyelocytes.
Hypersegmented neutrophils.

B
12
stores are big,

but folate stores are small.
Megaloblastic anemia is a
macrocytic anemia with oval macrocytes
and hypersegmented neutrophils.
Megal obl ast i c anemi a

The Complete (but not obsessive) Hematopathology Guide
page 20
www.pathologystudent.com
Hemolytic Anemias
Causes
I nheri t ed
Defects in red blood cell membrane (e.g., hereditary spherocytosis)
Enzyme deficiencies (e.g., glucose-6-phosphate dehydrogenase deficiency)
Defects in globin structure/synthesis (e.g., hemoglobinopathies)
Acqui red
Autoimmune hemolytic anemia
Microangiopathic hemolytic anemia

Cl i ni cal f eatures
Chroni c hemol yt i c anemi as
These are usually congenital, and are often well-compensated, with few symptoms. When
something happens to disturb the fragile equilibrium between red cell destruction and bone
marrow compensation, though (for example, infection with parvovirus B19), the already maxed-out
bone marrow cannot compensate, and the patient goes into a hemolytic crisis. Symptoms of such
crises include jaundice (from an increase in unconjugated bilirubin), splenomegaly, and gallstones.
Acut e hemol yt i c anemi as
These are usually acquired, and present comparatively suddenly with symptoms like backache,
abdominal pain, headache, malaise, fever, pallor, jaundice, and tachycardia.

Laboratory f i ndi ngs
Si gns of excessi ve red cel l dest ruct i on
Hemoglobinemia/hemoglobinuria (when hemolysis is intravascular, or so brisk extravascularly
that macrophages cannot keep up).
# serum unconjugated bilirubin (unless liver can keep up with excretion, in which case it will be
conjugated!).
# lactate dehydrogenase (LDH) (a red cell enzyme).
" haptoglobin (a protein that binds free Hgb).
Si gns of accel erat ed eryt hropoi esi s: ret i cul ocyt osi s
Reticulocyte count = percentage of red blood cells that are reticulocytes (normal=1-3%).
The lower the hemoglobin, the higher the reticulocyte count should be!
megaloblastic anemia
hypersegmented
neutrophil, oval
macrocytes
In hemolytic anemias, there is an
increased rate of red cell destruction,
and (hopefully) an increased rate
of red cell production
Ret i cul ocyt es

The Complete (but not obsessive) Hematopathology Guide
page 21
www.pathologystudent.com


Morphol ogy
Normochromic, normocytic (usually) anemia.
Spherocytes (found in almost every hemolytic process).
Other more specialized poikilocytes:
Target cells (thalassemias and hemoglobinopathies)
Elliptocytes (hereditary elliptocytosis)
Sickle cells (sickle cell anemia)
Schistocytes (microangiopathic hemolytic anemia)
Signs of increased erythropoiesis:
Polychromatophilia (or, if stained with supravital stain, reticulocytosis)
Basophilic stippling (RNA remnants not yet removed from cell)
Nucleated red blood cells (bone marrow hurrying to get red cells out as quickly as possible)

I mportant l ab test
Di rect ant i gl obul i n t est ( DAT)
Also called the Coomb's test. Mix patient's red cells with anti-human globulin (an antibody against
human immunoglobulins). If the patients red cells are coated with antibodies (as they are in some
immune processes, see later), the anti-human globulin will attach to those antibodies, bridging the
red cells and making them clump together. So, a positive result (red cell clumping) means the
patient's red cells are coated with antibodies, and the hemolysis is probably immune-related.

How to di agnose a hemol yti c anemi a
Fi rst , i s t here hemol ysi s?
Look for signs of increased red cell destruction (like increased bilirubin) and signs of increased
erythropoiesis (if hemolysis has been around long enough).
Second, what ' s causi ng t he hemol ysi s?
Using the history, DAT, and blood smear, you can categorize patients into five groups:
1. Patients with known exposure to infectious or chemical agents.
2. Patients with positive DAT (diagnosis: immune-related hemolytic anemia).
3. Patients with negative DAT but lots of spherocytes (probable diagnosis: hereditary spherocytosis).
4. Patients with negative DAT and other specific, morphologic abnormalities (e.g., sickle cells).
5. Patients with negative DAT and no specific morphologic abnormalities (do Hgb electrophoresis).

The Complete (but not obsessive) Hematopathology Guide
page 22
www.pathologystudent.com


Hereditary Spherocytosis

HS is a relatively common hemolytic anemia (1 in 5000 people of northern European descent) with
a variable age of onset and severity. Patients often (but not always) have a classic triad of mild
anemia, intermittent jaundice and splenomegaly. Anemia is exaggerated during crises, which are
frequently precipitated by infection with parvovirus B19 (fifth disease).

Pathogenesi s
The basic defect is in a protein called spectrin, which helps anchor the red cell membrane to the
cytoskeleton. Patients with HS have defective spectrin, which leads to altered membrane
properties (loss of surface area, altered membrane lipids/proteins) and the formation of
spherocytes (which get eaten up by macrophages in the spleen).

Morphol ogy
Mild normochromic, normocytic anemia.
Numerous spherocytes.
Evidence of accelerated hematopoiesis (polychromatophilia, normoblasts).

Treatment
Splenectomy cuts way down on the symptoms (because thats where the red cells get destroyed).
If splenectomy isnt possible, the patient may need red cell transfusions during crises.

Other hemol yti c anemi as due to i nheri ted membrane abnormal i ti es
Heredi t ary el l i pt ocyt osi s
Patients with HE have a spectrin abnormality too, but the mutation is different than that in HS. The
clinical features are similar to those in HS, but the blood smear shows tons of elliptocytes.
Heredi t ary pyropoi ki l ocyt osi s
In this disorder, there is a different problem with spectrin: it won't stick to itself, and therefore gets
degraded too rapidly. The clinical picture is pretty similar to HS and HE, but the morphology is
different: the red cells take on all kinds of bizarre shapes. They look like red cells that have been
exposed to heat thus the name "pyro".

Hereditary spherocytosis:
Lots of spherocytes due to
a spectrin defect.

Heredi tary spherocytosi s

The Complete (but not obsessive) Hematopathology Guide
page 23
www.pathologystudent.com
Glucose-6-Phosphate Dehydrogenase Deficiency

Glucose-6-phosphate dehydrogenase deficiency is a type of anemia that only appears after
exposure to certain oxidizing agents, like fava beans, or certain medications. Following exposure,
the red cells appear to have bites in them, from removal of inclusions called Heinz bodies.

Pathogenesi s
G6PD is a pretty important enzyme. It catalyzes the initial step in the pentose phosphate pathway
of glycolysis. Plus, you need G6PD to reduce NADP to NADPH, which in turn keeps glutathione in
the reduced state (reduced glutathione detoxifies hydrogen peroxide and other organic peroxides).
You normally make free radicals during routine cell life. When you get exposed to certain oxidizing
agents, though, you make a ton more. When there isnt enough G6PD around, these free radicals
attack sulfhydryl groups and break disulfide bonds in the cell. Heme is liberated from globin, and
globin is denatured, making a little round inclusion called a Heinz body which sticks to the red cell
membrane and compromises membrane plasticity. The red cells are detained in their passage
through the liver and spleen, where macrophages remove Heinz bodies, leaving a little bite in the
red cell.
The gene for G6PD is on the X chromosome. Therefore, males with G6PD deficiency usually have
full disease expression, and heterozygous females are clinically normal. G6PD deficiency is more
common in certain populations (10% of black men in US have the gene). The highest incidence is
in populations in which malaria has been endemic. It may confer a protective advantage against
malaria because G6PD-deficient red cells lack the ribose derivatives bugs need to grow.

Morphol ogy
Without exposure to offending agents, most patients have no anemia. After exposure, though,
patients get an acute hemolytic episode, with cell fragments, microspherocytes, and bite cells
(caused by recent pitting of Heinz bodies). Supravital staining reveals Heinz bodies (these
decrease in number as Hgb bottoms out, because younger cells have greater G6PD activity).

Treatment
In most affected individuals, severe hemolysis occurs only after exposure to oxidizing agents (such
as antimalarial drugs, sulfonamides, H
2
O
2
, aspirin, and fava beans) or infection (by unknown
mechanisms). So its important to avoid exposure to known oxidants. Usually the hemolysis is self-
limiting, with spontaneous resolution in a week or so, and no specific treatment is required.
Bi t e cel l
Without G6PD,
and Heinz bodies form.
free radicals accumulate

The Complete (but not obsessive) Hematopathology Guide
page 24
www.pathologystudent.com

Sickle-Cell Anemia

Sickle-cell anemia is one of a group of inherited disorders called hemoglobinopathies in which
structurally abnormal hemoglobin is made. Some of these abnormal hemoglobins function like
normal hemoglobin, but some have abnormalities that lead to hemolysis. Hgb S is the most
common of these; Hgb C is another fairly common one. Patients with two Hgb S genes are said to
have sickle-cell anemia, a disease characterized by hemolysis and microvascular occlusion.

Pathogenesi s
Most hemoglobinopathies are caused by a point mutation in a $ chain gene. Weird how just one
little point mutation can cause such a catastrophic disease. In the case of sickle-cell disease, the
mutation leads to a substitution of valine for glutamate in position 6 in the $ chain molecule, which
results in an abnormal hemoglobin, Hgb S, with abnormal physical and chemical properties. The
big problem with Hgb S occurs during deoxygenation. As the Hgb S molecules give up oxygen,
they aggregate and polymerize, distorting the red cell into a sickle shape. After many such
deoxygenation episodes, the red cell becomes permanently sickle-shaped.
This is not a great fate for the red cell because sickles are not as deformable as normal biconcave
disk-shaped red cells. Normal red cells flow easily through small vessels, but sickles are much
more likely to get stuck and pile up on each other, occluding vessels and causing ischemic tissue
damage. In addition, sickles are more fragile than regular red cells, so theres an element of
chronic hemolysis going on too.

Cl i ni cal Features
Sickle cell disease occurs predominantly in blacks. Eight percent of blacks in US are heterozygous
(heterozygosity is called sickle cell trait and rarely leads to significant clinical or hematologic
manifestations); one black child in 600 is homozygous (homozygosity is called sickle cell disease
and the severity varies quite a bit from patient to patient, for unknown reasons).
Patients have chronic hemolysis (the typical hemoglobin ranges from 6-10). Vaso-occlusive
disease usually occurs as an acute crisis, precipitated by infection, hypoxia, or other unidentified
causes. In children, pain in the hands or feet is often the first symptom. In all ages, bone, lung, and
abdominal pain is common.
Sickle cell anemia is a
qualitative abnormality
of hemoglobin

The Complete (but not obsessive) Hematopathology Guide
page 25
www.pathologystudent.com
The spleen may undergo massive enlargement (due to red cell sequestration) in early childhood.
By early adulthood, however, the spleen is reduced to a small, fibrotic remnant (due to recurrent
hemorrhage and fibrosis); this is called autosplenectomy. Because the spleen is so critical in
fighting off encapsulated organisms, its important to vaccinate patients with sickle-cell disease
against bugs like Streptococcus pneumoniae and Haemophilus influenzae.

Morphol ogy
In the blood, particularly during crises, sickle cells are present. After autosplenectomy occurs,
there is whats called a "post-splenectomy blood picture": nucleated red blood cells, targets,
Howell-Jolly bodies, Pappenheimer bodies, and a slightly increased platelet count (the platelets
love to hang out in the spleen, so when you take away their little home, they have no choice but to
hang out in the blood).

Treatment
Treatment is tailored to each patient. Its important to prevent triggers (which are different in
different patients, but include things like infection, stress, fever, dehydration, and hypoxemia). Extra
precautions must be taken to prevent infections due to encapsulated bugs (vaccinate, give
prophylactic antibiotics). Blood transfusions might be necessary during severe crises (maybe also
regularly, in certain patients). As an absolute last resort, bone marrow transplantation can be used
(but it has a relatively high mortality, so its not for everyone).


Si ckl e-cel l anemi a

The Complete (but not obsessive) Hematopathology Guide
page 26
www.pathologystudent.com


Thalassemia

The thalassemias are characterized by a quantitative decrease in one of the hemoglobin chains.
In %-thalassemia, there is a decreased amount of % chain. In $-thalassemia, there is a decreased
amount of $ chain. You end up with a two-fold problem:
1. Decreased hemoglobin production (because of the decrease in globin chains)
2. Excess unpaired % chains (in $ thalassemia) or $, &, and ' chains (in % thalassemia), which
form tetramers and lead to premature red cell destruction.
Lets look at this in a little more depth.

Pathogenesi s
Normal hemogl obi n composi t i on
Before 6 months of age, most of a persons circulating hemoglobin is fetal hemoglobin (HgbF),
which has two % chains and two & chains (%
2
&
2
). Around 6 months of age, hemoglobin composition
starts switching to the adult pattern, which is a composite of three types: 96% is HgbA (%
2
$
2
), 3%
HgbA
2
(%
2
'
2
), and 1% HgbF (%
2
&
2
). The genes for the %

and $ chains differ in number; there are four
%-chain genes and two $-chain genes.

Di sease severi t y depends on how many genes are def ect i ve ( or absent ) .
In $-thalassemia, the defect is in transcription, translation, or processing of mRNA. The defective $
gene is designated differently depending on the severity of the defect. A normal $ chain gene that
produces normal amounts of $ chains is designated a $ gene. A gene that produces no $ chains is
designated as $
0
, and a gene that produces some $ chains (but less than a normal amount) is
designated as $
+
.
So, depending on what kinds of genes you have, you can have anything from asymptomatic $
thalassemia to really severe $ thalassemia. (Notice that $
0
/$ appears twice in the chart below!
Thats because there is a variable clinical picture; not everyone with the same genetic defect has
exactly the same symptoms.)

Thalassemia is a
quantitative abnormality
of hemoglobin

The Complete (but not obsessive) Hematopathology Guide
page 27
www.pathologystudent.com







In %-thalassemia, the "defect" is really gene absence, so there are no different symbols or
designations for different kinds of % genes. You just put a little dash to indicate a missing gene.
Fortunately, there are four % genes so you can delete one or even two of them and still have a
normal life.







So to summari ze:
%-t hal assemi a
In %-thalassemia, the problem is deletion of one or more gene. The anemia is caused by two things:
1. Insufficient % chains (leading to insufficient Hgb A)
2. Excess unpaired $, &, and ' chains, which form tetramers. Newborns have higher % of
HgbF, so they make &
4
tetramers (called Hb Barts), whereas adults have mostly HgbA, so
they make $
4
tetramers (called HbH). Macrophages in the spleen gobble up these
tetramer-laden red cells. Yummy.
$-t hal assemi a
In $-thalassemia, the problem is in transcription, translation, or processing of mRNA. The anemia is
caused by two things:
1. Insufficient $ chains (leading to insufficient Hgb A)
2. Excess unpaired % chains (leading to premature red cell destruction because splenic
macrophages see these as yummy)
i f genot ype i s: di sease i s:
$
0
/$
0
or $
0
/$
+
$-thalassemia major (severe)
$
0
/$ or $
+
/$
+
$-thalassemia intermedia (less severe)
$
0
/$ or $
+
/$ $-thalassemia minor (mild or asymptomatic)
i f genot ype i s: di sease i s:
--/%% or -%/-% %-thalassemia trait (asymptomatic)
--/-% HbH disease (severe)
--/-- Hydrops fetalis (fatal in utero)

Things to make you look smart
Q. Bot h I DA and t hal assemi a are
hypochromi c and mi crocyt i c. So
how can you t el l t hem apart
morphol ogi cal l y?
A. There are two morphologic clues:
1. In IDA, there is usually a fair amount
of anisocytosis (which, if you recall from
our discussion of lab tests in the
beginning, is reflected in the RDW).
Most cases of thalassemia (except for
the really severe ones) show a very
minimal amount of anisocytosis. In fact,
the cells are virtually all the same exact
size! Weird. So: IDA has an increased
RDW, and thalassemia has a normal (or
decreased) RDW.
2. In IDA, the RBC is low (which makes
sense, because theres not enough iron
around to make red cells properly).
Most cases of thalassemia, however,
have an increased RBC, for some
unknown reason! Weird. So: IDA has a
decreased RBC, and thalassemia has
an increased RBC.
Of course, these are just morphologic
clues. To really make an official
diagnosis, youd need to do iron
studies (to look for iron deficiency) and
hemoglobin electrophoresis (to look at
the types and amounts of hemoglobin
present).



The Complete (but not obsessive) Hematopathology Guide
page 28
www.pathologystudent.com

Morphol ogy
%-t hal assemi a
Patients with silent carrier state and %-thalassemia trait have no anemia. Patients with HbH
disease have moderate to severe anemia with a range of changes similar to those described for $-
thalassemia, below. Supravital staining reveals HbH inclusions.
$-t hal assemi a
Patients with $-thalassemia minor have a mild microcytic, hypochromic anemia. There is usually
some basophilic stippling (blue dots in the red cells), and target cells.
Patients with $-thalassemia intermedia and $-thalassemia major have a severe anemia (Hgb=3-6)
with a very abnormal-looking smear. The red cells show marked anisocytosis and poikilocytosis,
and there are usually a bunch of nucleated red blood cells (as the marrow is trying hard to pump
out any red cells it makes as soon as it makes them).

Cl i ni cal f i ndi ngs
%-t hal assemi a
%-thalassemia is more common in Asians and Blacks. The silent carrier state and %-thalassemia
trait produce no symptoms. HbH disease produces a moderately severe anemia, and hydrops
fetalis is fatal in utero.
$-t hal assemi a
$-thalassemia is more common in Mediterraneans, Blacks and Southeast Asians. Patients with $-
thalassemia minor are almost always asymptomatic, with mild or no anemia. Patients with $-
thalassemia major have a very severe very severe, starting at 6-9 months of age. Without therapy,
children suffer growth retardation and die at a young age from profound anemia. Treatment
involves repeated transfusions (with iron chelation to avoid iron overload). Unless bone marrow
transplant is performed, many patients die in their 20s.

Treatment
Patients with mild thalassemia dont require treatment. Patients with severe anemia may need
repeated red cell transfusions or even bone marrow transplantation.
often have decreased anisocytosis

microcytic and hypochromic and

Thalassemias are

Moderat e t hal assemi a

The Complete (but not obsessive) Hematopathology Guide
page 29
www.pathologystudent.com
Warm Autoimmune Hemolytic Anemia

The autoimmune hemolytic anemias (AIHAs) are disorders in which the patient makes
autoantibodies against his or her red cells, leading to early red cell destruction. There are two
flavors of AIHA, warm and cold, so-named for the temperature at which these autoantibodies like
to react. This sounds arcane, but it really does matter, as well see below. If the antibody reacts at
a warm temperature, it can glom onto the red cell anywhere in the body, whereas if it reacts at
cool temperatures, it will bind to the red cell only in cool parts of the body and then fall off in warm
parts. This changes the way the disease presents. Lets start with warm AIHA.
Pathogenesi s
Most cases of warm AIHA are primary, or idiopathic (meaning: we have no idea what is causing it).
A smaller number of cases are secondary to things like leukemia or lymphoma, other malignancies,
autoimmune disorders, infections, or drugs (like methyl-dopa or penicillin).
For whatever reason, the patient makes IgG Ab that bind to his or her red cells. The binding
occurs best at 37 C, which just happens to be normal body temperature. Macrophages in spleen
think these IgG-coated (opsonized) red cells are yummy, and they nibble out bits of red cell
membrane, which makes the cells round up into spherocytes. Eventually, the macrophages eat up
the spherocytes entirely.
Cl i ni cal Features
Warm AIHA hits patients of any age or race, and either sex. Patients have a variably severe
anemia, and often there is some degree of splenomegaly (because of all the nom-noms).
Morphol ogy
The blood smear shows prominent spherocytosis and other signs of hemolysis (polychromasia,
normoblasts).
Di agnosi s
To make a diagnosis, you need to order a direct antiglobulin (Coombs') test (DAT), which detects
IgG and/or complement bound to red cells. Patients with warm AIHA are positive for IgG.
Treatment
Treat underlying cause, if there is one. Steroids can be useful, and if all else fails, splenectomy
might be necessary.
WAIHA:
IgG
spleen
spherocytes
Things to make you look smart
Q. Why i s t hi s coat i ng process
cal l ed opsoni zati on?
A. Opsonization comes from the
Greek word !"#$ (opson), which
describes the yummiest part of a
meal (like the relish that is eaten
alongside the more boring staple-
type food). This is a great word root
to use in this context, because things
that are opsonized (coated with
antibody or complement) look
particularly delicious to hungry
macrophages, who go after such
cells with gastronomic gusto.
Mmmmmmantibodylicious.

WAI HA

The Complete (but not obsessive) Hematopathology Guide
page 30
www.pathologystudent.com
Cold Autoimmune Hemolytic Anemia

Pathogenesi s
Just as in WAIHA, there are a bunch of possible etiologies in CAIHA. About half of the cases are
primary; the rest are secondary to infections (like mycoplasma pneumoniae or infectious
mononucleosis) or lymphoproliferative disorders (like lymphoma).
Patients make anti-red-cell antibodies that are IgM in nature (instead of IgG). For some reason,
they are usually directed against the "I" antigen on the red cell surface. These bind to the red cell
best at temperatures <37 C, which means they bind best in distal, colder body parts, but fall off in
warmer body parts. IgM generally exists as a pentamer, so when these antibodies bind, they end
up binding a bunch of cells together, causing agglutination!
In addition, complement binds to the red cells in these patients. Which leads to two things: 1) the
red cells get busted open, right in the circulation, and 2) macrophages in the liver and spleen
gobble up complement-coated red cells (but this type of hemolysis plays a smaller role than the
intravascular hole-poking type of hemolysis).

Cl i ni cal Features
Patients usually have a chronic, mild hemolytic anemia aggravated by the cold. Colder body parts
may look pale or cyanotic due to blockage of small vessels by red cell agglutinates.

Morphol ogy
If you make the blood smear at a cool temperature, you can see nice big red blood cell
agglutinates (clumps)! Spherocytes are present too, but theyre less numerous than they are in
WAIHA (in which the splenic biting process dominates). There are usually some other signs of
hemolysis, like polychromasia or normoblasts.

Di agnosi s
Just like in WAIHA, you need to do a DAT to make this diagnosis. The DAT doesnt detect IgM
antibodies, but fortunately it does detect complement. So if the test is positive for complement,
thats a good indicator that your patient has cold AIHA.

Treatment
Treat the underlying cause, if there is one. Its helpful to keep the patient warm. Splenectomy and
steroids are generally not helpful.
CAIHA:
IgM/complement
intravascular hemolysis
agglutination
CAI HA

The Complete (but not obsessive) Hematopathology Guide
page 31
www.pathologystudent.com


Microangiopathic Hemolytic Anemia

Microangiopathic hemolytic anemia (MAHA) is important not so much for the anemia itself, but for
what it may signify. There are a bunch of dangerous disorders that can cause MAHA so youll
want to be sure you can identify this anemia on a blood smear.

Pathogenesi s
The mechanism underlying microangiopathic hemolytic anemia is pretty straightforward: red cells
are ripped apart by physical trauma (either fibrin strands snag them or mechanical devices bash
them).

Causes
Narrowi ng/obst ruct i on of mi crovascul at ure
Disseminated intravascular coagulation (DIC)
This is a nasty condition in which patients get widespread clotting and bleeding (both!). Its seen in
many different settings in which the coagulation system is activated, like sepsis, abruptio
placentae, and disseminated malignancies.
Thrombotic thrombocytopenic purpura (TTP)
This syndrome consists of MAHA, thrombocytopenia, CNS dysfunction, renal failure and fever. Its
caused by a deficiency of an enzyme called ADAMTS13 (see box at right).
Hemolytic-uremic syndrome (HUS)
This syndrome has some overlap with TTP, in that patients get widespread thrombi and MAHA.
However, it occurs more often in children, and usually presents with bloody diarrhea and renal
failure. The pathogenesis is totally different than TTP; HUS is caused by infection with E. coli
0157:H7 (a toxin produced by the bug damages endothelial cells in the gut and kidney).
Malignant hypertension
In this nasty, high-mortality disorder, narrowed arterioles and turbulent blood flow can lead to red
cell fragmentation.
Systemic lupus erythematosus
Patients with SLE can get a necrotizing arteritis that leads to red cell fragmentation.
If a patient has MAHA,
you must find out why!
Things to make you look smart
Q. What causes TTP?
A. There is a big protein called von
Willebrand factor that helps platelets
stick down onto the subendothelium
and plug holes in vessels. It works
great, and we use it all the time when
we make clots.
However, its released in a really large
form, for some reason, which traps
platelets really really easily. Thats not
good. Theres an enzyme that takes
care of it, though, cleaving it into nice
little happy bits. That enzyme is called
ADAMTS13 (short for a big long word
that nobody better ever ask you to
repeat), and it does a great job of
keeping vWF in check.
You can imagine what would happen
if you didnt have enough
ADAMTS13: youd be making big
platelet clumps all over the place!
Thats exactly what happens in TTP.
Patients with TTP lack ADAMTS13
so they are unable to cleave vWF into
nice little chunks and they end up
with widespread platelet clumps. Not
good.


The Complete (but not obsessive) Hematopathology Guide
page 32
www.pathologystudent.com
Heart /great vessel abnormal i t i es
Okay, so these abnormalities arent really microangiopathic theyre macroangiopathic. They
still fall under the category of MAHA though. Artificial heart valves can lead to MAHA because
every time that artificial valve closes (especially the older ball-and-socket valves), a few red cells
get trapped in there and busted. Which doesnt sound like a lot, but if you consider how many
times the heart beats in an hour or a day thats actually a lot of red cells. Another
macroangiopathic cause of MAHA is coarctation of the aorta, a condition that creates unusually
turbulent (and red-cell-shearing) blood flow.

Morphol ogy
The blood smear in MAHA shows schistocytes, which is a fancy word for fragmented red cells.
You should not see any fragmented red cells in normal blood. If you do, you need to figure out
why. There are other disorders that have cells that can look kinda fragmenty (like the bite cells in
G6PD deficiency, for example). True schistocytes, however, are smaller than regular red cells. They
have points (sometimes they look like little triangles), and they lack central pallor. That being said, if
you have any doubt or question about whether a cell is really a schistocyte, you should err on the
side of being safe and work it up. Better safe than sorry.

Cl i ni i cal f eatures
Symptoms vary according to cause of MAHA. The important thing is usually not the anemia itself,
but the underlying disorder. Don't miss this one!
Schi stocyte

The Complete (but not obsessive) Hematopathology Guide
page 33
www.pathologystudent.com
I nf ecti ous condi ti ons
Pulmonary infections
Subacute bacterial endocarditis
Pelvic inflammatory disease
Osteomyelitis
Chronic urinary tract infection
Meningitis

I nf l ammatory condi ti ons
Rheumatoid arthritis
Systemic lupus
erythematosus
Severe trauma
Mal i gnant condi ti ons
Carcinoma
Lymphoma
Leukemia
Multiple myeloma




Anemia of Chronic Disease

Anemia of chronic disease is a mild to moderate anemia accompanying infections, inflammatory
disorders or malignant diseases that persist more than 1-2 months (see the boxes below). It's the
second most common anemia (after IDA) and it's characterized by low serum iron, despite lots of
macrophage storage iron.





Cl i ni cal Features
The symptoms are those of the underlying disease, not of the anemia. Anemia develops during the
first two months of the chronic disease, and doesn't progress thereafter.

Pathogenesi s
There is a complex disturbance in iron metabolism (iron never makes it into normoblasts). Mucosal
cells seem to absorb iron okay, but don't release it well into plasma. Further, macrophages take
up iron but release it very slowly. Recently, its been shown that patients with ACD have higher-
than-normal levels of something called hepcidin, a liver protein that regulates iron metabolism.
In addition, the red cells in ACD have a shortened survival. Weird fact: cells of patients with ACD
have normal lifespan when transfused into normal patients; but cells of normal patients have
shortened lifespan when transfused into patients with ACD. This probably has something to do
with increased phagocytosis of red cells by macrophages.



In ACD,
iron metabolism is disturbed
(iron doesn't get into normoblasts).

The Complete (but not obsessive) Hematopathology Guide
page 34
www.pathologystudent.com

Morphol ogy
Patients with ACD usually have a normochromic, normocytic anemia with minimal anisocytosis and
poikilocytosis. Meaning that when you look at the red cells under the microscope, they look
basically very normal. Some cases of ACD (about 25%) are microcytic, but the MCV rarely gets
below 72 fL.

I ron studi es
" serum iron
" TIBC (total iron binding capacity)
# ferritin (ferritin is an acute phase reactant! It goes up in the type of conditions
that cause ACD)
# bone marrow storage iron (not that youd do a bone marrow biopsy on a
patient with ACD!)

Di agnosi s
Youd suspect this anemia in a patient with a mild normochromic, normocytic anemia who has one
of the chronic diseases listed above. Iron studies would be confirmatory.

Treatment
ACD is usually so mild that no treatment of the anemia is required. The underlying disease is the
focus of the patients treatment.





ACD is bland-looking.
Anemi a of chroni c di sease

The Complete (but not obsessive) Hematopathology Guide
page 35
www.pathologystudent.com



Anemia of Chronic Renal Disease

Anemia of chronic renal disease is a narrow category of anemia that is due solely to decreased
erythropoietin. It may be complicated by iron deficiency anemia (secondary to hemodialysis),
anemia of chronic disease (secondary to concurrent chronic infection or inflammation), or
microangiopathic hemolytic anemia (secondary to changes in renal microvasculature) but the
pure form (just related to the decreased erythropoietin) is what were talking about here. Patients
are usually in the final stages of severe renal disease with all its attendant symptoms (anemia is
usually an incidental finding). A rough correlation exists between the severity of the anemia and the
degree of renal insufficiency.

Morphol ogy
This is a normocytic, normochromic anemia with minimal anisopoikilocytosis. The only really
unique finding is the occasional presence of echinocytes, or "burr" cells, which are red cells with a
bunch of short, spiky surface projections.

Pathogenesi s
Patients with bad kidney disease secrete less erythropoietin, a hormone that stimulates red cell
growth.

Treatment
The anemia is usually mild and requires no treatment. Eventually, patients with renal failure may be
treated with a kidney transplant (in which case, the anemia then corrects) or dialysis (which doesnt
do anything to erythropoietin levels). If renal failure is severe enough, and the patient will remain on
long-term dialysis, recombinant erythropoietin may be used.






In anemia of chronic renal
disease, you may see
echinocytes.
Echi nocyt es

The Complete (but not obsessive) Hematopathology Guide
page 36
www.pathologystudent.com



Anemia of Chronic Liver Disease

Anemia of chronic liver disease, in its pure form, is a mild to moderate anemia due to the
shortened red blood cell survival and impaired bone marrow response typically seen in patients
with chronic liver disease. Many other factors may contribute to anemia in patients with alcoholic
liver disease, such as folate deficiency (which leads to megaloblastic anemia) and hemorrhage
from upper GI varices or hemorrhoids (which leads to iron deficiency anemia). Pure,
"uncomplicated" anemia (without the above factors) is actually uncommon.

Pathogenesi s
The red cells in patients with chronic liver disease dont live as long (due, in part, to the congestive
splenomegaly that accompanies most cases). In addition, there is a decreased bone marrow
response to the anemia, for a couple reasons: ethanol inhibits erythropoiesis (and the cause of
many cases of chronic liver disease is alcoholism), and in addition, the small but significant amount
of erythropoietin secreted by the liver is diminished.

Morphol ogy
Pure cases of anemia in chronic liver disease (without complicating factors, like folate or iron
deficiency) usually manifest as a normochromic, normocytic anemia. Some cases are macrocytic
(MCV>100). Theres an increased amount of poikilocytosis, with target cells (in chronic liver
disease, there is increased cholesterol and lecithin in the red cell membrane, which leads to
increased cell surface area) and acanthocytes, or "spur" cells (red cells with 5-10 long, spiky
surface projections).

Cl i ni cal
Three-fourths of patients with liver disease are anemic! Interesting fact: alcohol abusers with even
mild liver disease can get episodes of hemolysis which resolve when alcohol is withdrawn. Makes
you think.


In anemia of chronic liver
disease, you may see
acanthocytes.

The Complete (but not obsessive) Hematopathology Guide
page 37
www.pathologystudent.com

Aplastic Anemia

Aplastic anemia is a disease in which the bone marrow is empty that is, there are very few, if
any, hematopoietic precursors. Its really more than just an anemia; its a pancytopenia (meaning
that all the major cell lines red cells, white cells, platelets are severely decreased).

Pathogenesi s
The list of possible causes is long but most of the time, no specific cause can be identified. Of
the remaining cases, most are acquired (meaning youre not born with it), and are due to things
like drugs (e.g., chloramphenicol), viruses (e.g., HIV), pregnancy, or radiation. A small number of
cases are familial. The most common of these familial causes of aplastic anemia is Fanconi
anemia, a hereditary disease characterized by skeletal abnormalities, chromosomal instability,
pancytopenia and increased risk of leukemia.

Cl i ni cal f eatures
Fortunately, aplastic anemia is a rare disease. The death rate is approximately 1-13
people/1,000,000 per year. It is not age- or sex-related. The signs and symptoms are related to
the pancytopenia. Patients are pale, and feel fatigued and dizzy (from the anemia); they may get
recurrent infections (from the leukopenia); and they may have excess bleeding or bruising (from the
thrombocytopenia).

Morphol ogy
Bl ood
The blood is pancytopenic, meaning that the red cells, white cells, and platelets are all decreased.
The anemia itself is bland its normochromic and normocytic with minimal anisopoikilocytosis.
Bone marrow
The bone marrow is markedly hypocellular, or "empty". There is a lot of fat, and a few
lymphocytes, but very very few red cell, white cell, and platelet precursors. The bone marrow
aspirate specimen consists mostly of lymphocytes, with some mature red cells but very few
hematopoietic precursors.

Patients with aplastic anemia
have pancytopenia
and an empty marrow.
Apl asti c anemi a: bone marrow

The Complete (but not obsessive) Hematopathology Guide
page 38
www.pathologystudent.com

Di f f erenti al di agnosi s
If all you have to go on is a pancytopenic blood smear (no bone marrow biopsy), then in addition
to aplastic anemia, youd have to consider disorders infiltrating the bone marrow (e.g.,
myelofibrosis, metastatic carcinoma), disorders involving the spleen (e.g., congestive
splenomegaly, lymphoma) and other miscellaneous disorders (e.g., overwhelming infection, and
something called refractory anemia which well mention when we get to myelodysplastic
syndromes). To get to the bottom of things, youd really need a bone marrow biopsy.

Treatment and prognosi s
Obviously, if the anemia is the result of exposure to a known noxious agent, it would be a good
idea to avoid further exposure. More specific treatment may include transfusion of blood
components as needed (red blood cells, platelets), drug therapy to stimulate hematopoiesis (G-
CSF, steroids, androgens) and suppress the immune system (anti-thymocyte globulin), and if
necessary, bone marrow transplant. The 5-year survival rate is around 70-90%.







The Complete (but not obsessive) Hematopathology Guide
page 39
www.pathologystudent.com


Three. Beni gn Leukocytoses

There are lots of benign reasons for an elevated white cell count (leukocytosis). There are, as you
know, several different kinds of white cells, each with its own unique characteristics and functions
so there are several different types of benign leukocytosis, each with its own causes and
implications. Well discuss these separately, with emphasis on neutrophilia and lymphocytosis.

Benign Neutrophilia

Where do neutrophi l s l i ve?
Most neutrophils live in the bone marrow. Only about 5% live in the blood. Heres a weird fact: half
of the neutrophils in the blood are marginated meaning that they are plastered up against the
walls of the vessels! This is important because if you get emotionally stressed (like, say, in the
emergency room, or during a particularly bad pathology exam), you can pull most of those
marginated neutrophils off the walls of the vessels and into the bloodstream, effectively doubling
your neutrophil count. Something to remember when you see a child in the emergency room with
a high neutrophil count.

Normal neutrophi l count
The normal range for neutrophils is between 2 and 8 x 10
9
/L. Like most tests with a reference
range, it varies a bit by institution or laboratory so youd need to check your exact range. There
are some physiologic variations in the neutrophil count. Hormones can change the neutrophil
count (it goes up in pregnancy and menstruation and down after menopause), as can the typical
medical or dental student trifecta of stress, smoking and alcohol (all of which raise the neutrophil
count). Theres some diurnal variation too, with counts being higher in the evening than in the
morning.


The Complete (but not obsessive) Hematopathology Guide
page 40
www.pathologystudent.com


Prol i f erati on of mature neutrophi l s
When you have a patient with a high neutrophil count, it pays to look and see what kind of
neutrophils are proliferating mature ones (segmented neutrophils, the ones you usually see in
peripheral blood), or immature ones (like bands, metamyelocytes, myelocytes, promyelocytes
cells you usually see only in the marrow). The causes of a mature neutrophilia are slightly but
significantly different than the causes of an immature neutrophilia.
If all you see are mature neutrophils, the main causes you should consider are infection (usually
bacterial, not viral) and inflammation. Sometimes, in infections, the neutrophils will show odd
changes called toxic changes. These include so-called toxic granulation (abundant, deep dark
granules in the cytoplasm), Dhle bodies (pretty, sky-blue blobs in the cytoplasm, probably
representing revved-up rough endoplasmic reticulum), and cytoplasmic vacuolization (just what the
name says). You can see these changes in any type of bacterial infection, but they are especially
abundant in serious infections like sepsis.

Prol i f erati on of i mmature neutrophi l s
If you see immature neutrophils, then its a different story. There are three different kinds of
immature neutrophilia: left shift, leukemoid reaction (bad term! bad term!), and
leukoerythroblastotic reaction.

Lef t shi f t
A left shift just means that there are immature neutrophils in the blood. The causes of a left shift
are fairly similar to the things we talked about above under mature neutrophilia, namely, infection
and inflammation.

Leukemoi d react i on
This is a bad term! Dont use it! The reason its bad is because it doesnt have a single, clear
definition. The word leukemoid means that when you look at the blood, it looks like leukemia but
it isnt. So that could mean that theres a very high neutrophil count, or it could mean that theres a
marked left shift, or it could mean bothsee how confusing it is? Youll hear some older
physicians use the term (and you might read it on some websites of questionable repute); youd
have to figure out what the actual morphologic changes are in order to make some sense out of it.
Or you could just walk away, shaking your head and smiling.
Lef t shi f t
Toxi c granul at i on ( L) and vacuol i zat i on ( R)

The Complete (but not obsessive) Hematopathology Guide
page 41
www.pathologystudent.com

Leukoeryt hrobl ast ot i c react i on
Now this is not a bad term. It has a very well-established definition: it means that there are both
immature neutrophil precursors and immature red cell precursors in the blood. By the way,
anything other than a regular old red cell would be considered immature in the blood. Any red cell
precursor with a nucleus, no matter what the stage of development, is normally seen only in the
marrow.
There is a different set of causes for this particular kind of immature neutrophilia, and not all of
them are nice causes. Sometimes, in situations when the hemoglobin is really low (like, say, under
6), the bone marrow is so bent on getting red cells out into the periphery that it ends up letting a
few nucleated red cells slip out along with all the mature red cells. The marrow is a little sloppy
sometimes, and on occasion, it might let some immature neutrophils out at the same time. In that
setting, a leukoerythroblastotic reaction (LEBR) would be considered physiologic, and you
wouldnt worry too much about it (other than youd want to know why the patient was so anemic).
In other settings, however, an LEBR is more ominous. Unless theres a damn good reason for the
bone marrow to be pouring immature hematopoietic cells into the blood (like, as we just
discussed, a very low hemoglobin), then what youd worry about is that there is something filling
up the marrow like carcinoma, or leukemia, or fibrosis that is pushing those precursors out into
the blood prematurely. Fully two-thirds of LEBRs occur in this second, scary scenario. Use the
hemoglobin to help differentiate between scary and non-scary causes. If the hemoglobin is below
6, an LEBR is probably physiologic; if its over 6, make sure there isnt something else going on.


Things to make you look smart
Q. Why i s a bunch of i mmat ure neut rophi l s i n t he bl ood cal l ed a l ef t shi f t ?
A. Its called a left shift because in the olden days, when people counted cells in the
blood, they didnt have any electronic gizmos; they just tallied them by hand in columns.
By convention, the earliest precursors (myeloblasts) were listed on the left, and then the
rest of the precursors in order after that, with segmented neutrophils farthest on the right.
So in a normal blood smear, all the marks would be in the right-most column, but if there
were immature neutrophils present, the marks would get shifted to the left.


LEBR + Hgb<6: probably okay.
LEBR + Hgb>6: could be bad

The Complete (but not obsessive) Hematopathology Guide
page 42
www.pathologystudent.com



Benign Lymphocytosis

Where do l ymphocytes l i ve?
Most lymphoid stem cells in adults are in the bone marrow. Growth factors like IL-2 induce
differentiation into T, B, and NK cell precursors, which then travel to different organs (e.g., lymph
nodes, spleen, thymus, mucosa-associated lymphoid tissue) for further processing and storage.

Normal l ymphocyte count
Unlike the normal neutrophil count, which is pretty much the same no matter how old you are, the
normal lymphocyte count varies with age. Its highest right after birth (at 2 weeks: 2.0 - 17.0 x
10
9
/L), and declines with age (at age 4: 2.0 - 8.0 x 10
9
/L; by age 18: 1.0 - 4.0 x 10
9
/L). Another
important parameter is the immunophenotype of the lymphocytes. Normally, we have way more T
cells than anything else (about 60-80% of our lymphocytes are T cells; 10-20% are B cells, and 5-
10% are NK cells). That can be useful in telling a benign lymphocytosis from a malignant one,
because cells in lymphoid malignancies are all the same type (all B cells or all T cells).

Prol i f erati on of mature l ymphocytes
Benign lymphocytosis has two flavors: mature and reactive. If all you see are a bunch of mature-
appearing lymphocytes, then the top three benign choices are infectious lymphocytosis, whooping
cough, and transient stress lymphocytosis. Infectious lymphocytosis is a viral disorder of childhood
in which the lymphocyte count can climb as high as 100 x 10
9
/L. Whooping cough (caused by
Bordetella pertussis infection) usually shows a moderate lymphocytosis (up to 55 x 10
9
/L).
Transient stress lymphocytosis is a temporary phenomenon seen in adults with emergency
medical conditions, such as cardiac arrest. The count generally does not exceed 10 x 10
9
/L, and it
generally falls back to normal within a day or two.

Mature lymphocytosis:
infectious lymphocytosis,
Bordetella pertussis, or
stress lymphocytosis
Mat ure l ymphocyt osi s i n
Bordet el l a pert ussi s i nf ect i on

The Complete (but not obsessive) Hematopathology Guide
page 43
www.pathologystudent.com

Prol i f erati on of reacti ve l ymphocytes
If, on the other hand, the lymphocyte count is elevated and you see lots of funny-looking
lymphocytes, you may be dealing with a reactive lymphocytosis. There are several kinds of
reactive lymphocytes, which are simply benign lymphocytes that have a different appearance
than the normal, mature lymphocytes you typically see in the blood.
One group of reactive lymphocytes is termed Downey cells, after the hematologist Hal Downey
who described them in 1923. Downey described three types of cells commonly found in infectious
mononucleosis: a small lymphocyte with lobed nucleus and scant, compact cytoplasm (later
termed the Downey I cell); a large, fried-egg lymphocyte with copious cytoplasm containing radial
striations (later termed the Downey II cell), and a very large lymphocyte with a large nucleus
showing a fine chromatin pattern (later termed the Downey III cell). Downey cells are seen in
infectious mononucleosis but also in pediatric viral infections such as measles, mumps, rubella and
chickenpox.
Other reactive lymphocytes include plasma cells and their precursors (like proplasmacytes, cells
that are almost to the plasma cell stage, but dont yet have nice clock-face chromatin or a well-
developed hof), and plasmacytoid lymphocytes (cells that have an eccentric nucleus and a hof, like
plasma cells, but with clumpy-smudgy lymphocyte chromatin rather than the blocky clock-face
chromatin of a plasma cell). These reactive lymphocytes are seen in pediatric childhood infections,
viral hepatitis, and various immune disorders such as autoimmune diseases and drug reactions.








Things to make you look smart
Q. What s t he si gni f i cance of t he di f f erent t ypes of Downey l ymphocyt es?
A. Of the three types of Downey cells, the one most commonly seen in infectious
mononucleosis is the type II cell (the one that looks like a fried egg). However, you can see this
cell in other benign viral disorders too. The type of Downey cell that is the most specific for
infectious mononucleosis is the type III cell (the huge one with fine chromatin). If you see this
one, you can be pretty darn sure that the patient has mono.


Propl asmacyte ( L) and
Downey I I I cel l ( R)

The Complete (but not obsessive) Hematopathology Guide
page 44
www.pathologystudent.com

Differentiating Benign from Malignant Leukocytoses
In real life, a patient does not come to you and say, I have a benign lymphocytosis. Please tell me
what the cause is. What happens is that you get a blood smear, and you have to make sure its
not malignant (then you can go about your business of figuring out which benign disorder youre
dealing with). So lets talk about how you would differentiate between these benign things and a
couple malignant things that could look very similar.
Lef t shi f t vs. CML
The left shift that we talked about can look very similar to a malignant disorder known as chronic
myeloid leukemia (or CML), discussed on page 65. Patients with CML have a very high white count
in the blood, and virtually all the cells are neutrophils in varying stages of development. So how
would you tell the difference between the two?
Well, patients with CML generally have more immature cells (like myelocytes and promyelocytes)
than patients with a benign left shift (in which you see a lot of bands and metamyelocytes, but
fewer of the earlier precursors like promyelocytes). In other words, the shift to the left is usually
more extreme in CML than it is in a benign left shift. Also, the white count itself is usually higher in
CML (on the order of 50,000 100,000) than it is in a benign left shift (where the count doesnt
usually go above 25,000 or so).
Those are pretty soft criteria, though. Something more concrete is basophilia, a finding that is
virtually always present in CML, for some weird reason. Finally, if you really want to be definitive, all
cases of CML have a chromosomal translocation between chromosomes 9 and 22, producing
whats known as the Philadelphia chromosome. Benign left shifts do not have this.
Mature l ymphocytosi s vs. CLL
In the lymphoid category, the benign mature lymphocytosis we talked about earlier can look
identical to a malignant lymphoid disorder called chronic lymphocytic leukemia (or CLL), discussed
on page 72. Patients with CLL have an elevated lymphocyte count, and all the lymphocytes look
mature. In fact, if all you had was a blood smear of each disorder, youd be very hard pressed to
tell them apart. Whats a person to do?
One thing to do would be to look at the age of the patient, if you have that information. Most cases
of mature lymphocytosis occur in kids, and all (yes, you read that right, all) cases of CLL occur in
adults. Another thing to do would be flow cytometry. In a benign mature lymphocytosis, there
would be a mixture of B and T cells, but in CLL, all the cells mark as B cells (and, strangely, they
also express CD5, a marker traditionally thought of as a T cell marker).
CML vs. left shift:
Philadelphia chromosome
CLL vs. mature lymphocytosis:
flow

The Complete (but not obsessive) Hematopathology Guide
page 45
www.pathologystudent.com

Other Leukocytoses

Monocytosi s
The normal monocyte count is somewhere around 0.3 - 0.5 x 10
9
/L. An elevated monocyte count
may be associated with solid malignancies, autoimmune disease, and infection.

Basophi l i a
The normal basophil count is between 0.01 - 0.1 x 10
9
/L. There is only one real cause of
basophilia, and thats chronic myeloid leukemia. You may see iron-deficiency anemia as a cause in
board review books, but theyre full of crap (at least as far as basophilia goes). Any time you see a
basophilia, you must rule out CML, even if the patient has no other symptoms or signs, because
basophilia is one of the earliest signs of CML!

Eosi nophi l i a
The normal eosinophil count is 0.05 - 0.3 x 10
9
/L. Some fairly frequent causes of eosinophilia
include drug allergies, asthma, and skin diseases. Rarely, a patient with eosinophilia will be found
to have intestinal parasitism (but for the frequency of test questions on this particular fact, youd
think it happened all the time!). Other rare causes include chronic ulcerative colitis, hepatitis and
sarcoidosis .


Monocyt osi s

The Complete (but not obsessive) Hematopathology Guide
page 46
www.pathologystudent.com
Four. Leukemi a

Introduction to Hematologic Malignancies

Before we get into our discussion of leukemia, lets take a moment to review how hematopoietic
malignancies are organized.

Leukemi a vs. l ymphoma
There are two main types of hematopoietic malignancies: leukemias and lymphomas. Leukemias
are hematopoietic malignancies that begin in the bone marrow. They almost always spill over into
the blood, and sometimes they involve other tissues too, like lymph nodes, spleen or CNS.
Lymphomas are hematopoietic malignancies that start in lymph nodes. They may spill over into the
blood, and sometimes they involve other tissues, like bone marrow, spleen or CNS.
Leukemias and lymphomas have their own classification schemes, and they are discussed
separately. However, there are many leukemia/lymphoma pairs that are now known to be the
same diseases. For example, chronic lymphocytic leukemia is now known to be the same disease
as small lymphocytic lymphoma (both have the same morphology, immunophenotype and
prognosis), and they are now lumped together as CLL/SLL. Well point out these pairings as we
go along.

Myel oi d vs. l ymphoi d
Besides the leukemia/lymphoma division, there is another big division in hematopoietic
malignancies, namely, that between lymphoid and myeloid malignancies. Lymphoid cells include
lymphocytes and their precursors. Myeloid cells include, well, everything else (neutrophils,
eosinophils, basophils, monocytes, red cells, and platelets).
By the way, the word myeloid means of the marrow so basically everything that grows and
lives in the marrow (that is, every hematopoietic cell except lymphocytes) is considered myeloid.
Just to make things more interesting, sometimes people use the term myeloid to refer to cells of
the neutrophil lineage. Confusing, isnt it? So if someone says myeloid, you need to figure out
whether theyre talking about all non-lymphoid cells or just neutrophils.
Leukemia:
bone marrow
Lymphoma:
lymph nodes

The Complete (but not obsessive) Hematopathology Guide
page 47
www.pathologystudent.com


Summari zi ng the di seases i n your head
Heres a couple of ways you may want to group these diseases together:



























The Leukemi a-vs.-Lymphoma Groupi ng
Leukemi as
Acut e l eukemi as
Acute myeloid leukemia
Acute lymphoblastic leukemia
Chroni c l eukemi as
Chronic myeloproliferative disorders
Chronic lymphoproliferative disorders
Myel odyspl ast i c syndromes

Lymphomas
Non-Hodgkin lymphoma
Hodgkin disease

Pl asma cel l di sorders
Myeloma
The Myel oi d-vs.-Lymphoi d Groupi ng
Myel oi d mal i gnanci es
Acute myeloid leukemia
Chronic myeloproliferative disorders
Myelodysplastic syndromes

Lymphoi d mal i gnanci es
B-cel l mal i gnanci es
Acute lymphoblastic leukemia, B-cell type
Non-Hodgkin lymphoma, B-cell types
Myeloma
T-cel l mal i gnanci es
Acute lymphoblastic leukemia, T-cell type
Non-Hodgkin lymphoma, T-cell types
Hodgki n di sease

The Complete (but not obsessive) Hematopathology Guide
page 48
www.pathologystudent.com
Acute Leukemias
As youll see in the next two sections, there is a big conceptual difference between the acute and
chronic leukemias. They are very different clinically (acute leukemias have a sudden, tumultuous
onset; chronic leukemias present more insidiously) and morphologically (acute leukemias have tons
of blasts; chronic leukemias have more mature cells). It will be helpful if you keep these
generalizations in mind as we talk about the different types of leukemia. Otherwise it can be easy
to get lost in the trees and miss the forest.

Pathophysi ol ogy
The acute leukemias are malignant, monoclonal proliferations of immature myeloid or lymphoid
cells in the bone marrow. Thats the official definition. If you want a quick and dirty definition, its
when the marrow (and usually the blood) fills up with malignant blasts. The process is caused by
two things: clonal expansion (one crappy little cell divides into two, and those divide into four, and
then 16, and then 32, etc. etc. until you have a huge population of cells all descended from one
bad little cells) and maturation failure (where instead of growing and maturing, cells stay stuck at
one stage of development).
Its not a great idea to have these malignant cells around for many different reasons. One big
problem is something called bone marrow failure. If you have a tone of cells of any type filling up
the marrow space, then theres not going to be much room for normal red cells, white cells and
platelets to grow so their counts will go down in the blood, and there wont be enough of them
to do their respective jobs. Sometimes, the malignant cells themselves can release factors that
inhibit growth and function of normal hematopoietic cells. Finally, if the malignant cells leave the
bone marrow space, they can infiltrate and damage other organs (liver, spleen, brain, others).

Cl i ni cal Features
Acute leukemias come on suddenly, over a period of days or at most weeks. Usually, the
presenting symptoms are related to bone marrow failure. Patients get fatigued and tachycardic
(from the anemia); they may have recurrent infections (from the decreased numbers of normal
white cells); and they may have excessive bleeding or bruising (from the thrombocytopenia). Some
patients also get bone pain (from the rapidly-expanding mass of malignant cells in their bones), or
symptoms of extramedullary involvement (like lymphadenopathy, hepatosplenomegaly, or CNS
infiltration).
few or no normal hematopoietic
cells in bone marrow
Bone marrow failure:
Bl ood f ul l of bl ast s i n acut e l eukemi a

The Complete (but not obsessive) Hematopathology Guide
page 49
www.pathologystudent.com
Acute Myeloid Leukemia
Acute myeloid leukemia (AML) is a malignant proliferation of myeloid blasts (or very early myeloid
precursors) in the bone marrow and blood. By the way, myeloid here means anything but
lymphoid. So the myeloid blasts might be myeloblasts (that would turn into neutrophils), or
monoblasts, or erythroblasts, or megakaryoblasts.
The blasts (or very early myeloid precursors) must comprise at least 20% of all the nucleated cells
in the blood and/or bone marrow to make the diagnosis of AML (if only 10% of all the nucleated
cells are blasts, for example, then you cant call it AML).

Cl assi f i cati on
There are lots of different kinds of AML. Some have unique genetic abnormalities. Some have
monoblasts, or erythroblasts, or something else instead of myeloblasts. Some occur as a result of
previous chemotherapy. All of these different types have to be classified in some way that makes
sense for therapy and prognosis.
Right now, were transitioning between two different classifications. The old classification (called
the FAB, or French-American-British classification) was based mostly on morphology (how the
cells look). It labels the different types of AML with numbers (AML-M0, AML-M1, AML-M2, etc.).
The new classification (created by the World Health Organization), takes into account a lot of
different things, such as morphology, genetic changes, and whether the leukemia is related to
previous chemotherapy. The new WHO classification is more helpful than the old FAB one for
determining treatment and prognosis. However, youll definitely see references to the old FAB
classification from time to time.

WHO Cl assi f i cati on of Acute Myel oi d Leukemi a
1. AML with genetic abnormalities
t(8;21)
inv(16)
t(15;17)
11q23
2. AML with an FLT3 mutation
3. AML with multilineage dysplasia
4. AML and myelodysplasia, therapy-related
5. AML not otherwise characterized
need at least 20% blasts
To make a diagnosis of AML,

The Complete (but not obsessive) Hematopathology Guide
page 50
www.pathologystudent.com
How to Make a Di agnosi s
Morphol ogy
In AML, youll see a lot of blasts (or very early myeloid precursor cells) in the blood and bone
marrow. You have to count them, though, because theres a cutoff you need to make in order to
call it AML: at least 20% of all nucleated cells (that means everything except mature red cells) in
the blood or bone marrow must be myeloblasts.
Some AMLs, as well see in a minute, dont have a ton of myeloblasts, but instead have a ton of
some other really early myeloid precursor. In acute promyelocytic leukemia, for example, the
proliferating malignant cell is the promyelocyte (there arent that many myeloblasts around). In
those types of AML, that particular early precursor cell is substituted for the myeloblast when
determining whether the case makes the 20% cutoff. So for acute promyelocytic leukemia, if more
than 20% of the nucleated cells in the blood or bone marrow are promyelocytes, thats enough to
call it acute promyelocytic leukemia (you dont need to worry about myeloblasts).
In case you need a little refresher as to what different cells look like, heres a list of the cells that
can be more difficult to distinguish:
Myel obl ast Large nucleus with fine chromatin, nucleoli. Thin rim of basophilic
cytoplasm. May have a few fine, azurophilic, cytoplasmic granules.
Promyel ocyt e Biggest cell in myeloid line.
Lots of primary (coarse, azurophilic) granules.
Monobl ast Typical blast cell but larger, with more oval-shaped nucleus.
Promonocyt e Delicate "tissue-paper" folds in nucleus.
Monocyt e Large cell with grayish ("dishwater") cytoplasm, "raked" chromatin.
In some cases of AML, there is pretty much nothing but blasts. Everywhere you look, just blasts;
no differentiating cells at all. Those can be difficult to diagnose. How do you even know if its a
myeloid leukemia? There are a couple of morphologic clues that will help you place the leukemia in
the myeloid category: dysgranulopoiesis and Auer rods
Dysgranulopoiesis means disordered neutrophil production (translation: funny-looking
neutrophils). Usually, even if you have a sea of blasts, if you look long enough you can find a
mature neutrophil here and there. If those neutrophils are strange-looking, thats a clue that the
blasts are myeloid. Sometimes the neutrophils will not have enough granules in the cytoplasm
(hypogranularity); sometimes instead of multiple nuclear lobes, youll just see two, or even one
(hyposegmentation).

The Complete (but not obsessive) Hematopathology Guide
page 51
www.pathologystudent.com

A more definitive clue is the presence of Auer rods. Auer rods are needle-like, eosinophilic
inclusions in the cytoplasm of malignant myeloblasts. They are composed of secondary granules
lined up all in a row but they look more like toothpicks than collections of granules. Auer rods are
only seen in malignant cells of the neutrophil series (and virtually always, they occur at the
myeloblast stage of development). So if you find a blast in that sea of blasts that has an Auer rod
in it, you know for sure that a) it is a myeloblast, and b) it is malignant. And by extension, the rest of
the blast population is myeloid as well. A note of caution: just because you dont see Auer rods,
that doesnt mean you can rule out AML! There are lots of cases of real AML in which you dont
see Auer rods. So: if you see them, its AML. If you dont see them, it could be either AML or ALL.

Cyt ochemi cal st ai ns
Sometimes, you have a ton of blasts, and no dysgranulopoiesis and no Auer rods. Then what?
Well, youll probably be sending the specimen off for other tests (like Immunophenotyping and
cytogenetics). But those tests take a while to perform. In the meantime, there are a few stains you
can do in the lab, right on an unstained smear, that can be very helpful.
Myeloperoxidase (MPO) is a stain that highlights neutrophils and their precursors (it also highlights
eosinophils, but a leukemia of eosinophilic cells is exceedingly rare, so this isnt usually an issue).
So, if you have a ton of blasts, and they light up with myeloperoxidase staining, you know they are
myeloblasts. Sudan black B (SBB) does the same thing; its just not used as frequently as MPO.
Another commonly-used cytochemical stain is non-specific esterase (NSE). This stain highlights
monocytes and their precursors. So the same reasoning applies: if you have a ton of blasts, or
blast-like cells, and they light up with NSE, you know they are of the monocytic lineage (probably
either monoblasts or promonocytes).

I mmunophenot ypi ng
As mentioned on page 11, immunophenotyping takes a look at the markers on the surface of the
cells. This is of great use when you dont know what a cell is! Common myeloid markers include
pan-myeloid markers such as CD33 (that stain virtually all myeloid cells) and more specific
myeloid markers, such as CD61 (which stains only megakaryoblasts). One thing to be aware of,
though, is that acute myeloid leukemias can sometimes aberrantly express lymphoid markers. Just
to make things more difficult.
Bl ast wi t h t wo Auer rods
Monobl ast s and a monocyt e ( L)
posi ti ve f or NSE

The Complete (but not obsessive) Hematopathology Guide
page 52
www.pathologystudent.com
Cyt ogenet i cs/Mol ecul ar St udi es
Several subtypes of AML have specific chromosomal abnormalities. Compared to other types of
AML, these subtypes are treated differently and have different prognoses. The most important
abnormalities are:
t(8;21) (better prognosis)
t(15;17) (better prognosis)
inv(16) (better prognosis)
11q23 (worse prognosis)
AML with FLT3 mutation (worse prognosis)
The only way to detect these, of course, is to do cytogenetic or molecular assays.

AML wi th geneti c abnormal i ti es
Now lets take a look at the different types of AML. Well use the WHO classification as our overall
guide, but well also throw in the FAB subtypes when appropriate. That way youll have heard of
everything, and youll be super smart no matter what classification people throw at you.
The first subtype is AML with genetic abnormalities. Cases of AML in this category have you
guessed it specific genetic abnormalities. These abnormalities confer either a better or worse
prognosis, and the leukemias are treated with different drugs. There are only four abnormalities in
this category: t(8;21), t(15;17), inv(16), and 11q23. Mutation of the FLT3 gene is also very
significant but that genetic abnormality gets its own category.

AML wi t h t ( 8; 21)
Cases of AML with a t(8;21) usually have the morphology of an AML-M2 in the old FAB
classification (a lot of myeloblasts, with some maturing neutrophilic cells too). Prognosis is relatively
favorable.

AML wi t h i nv( 16)
Cases of AML with an inv(16) usually have the morphology of an AML-M4 in the old FAB
classification (lots of myeloblasts and neutrophil precursors, but also lots of monocytic cells). In
addition, there are some unusual morphologic findings in these cases most notably, the
presence of huge eosinophils with deep purple granules (weird). These cases are sometimes
called AML-M4 Eo because of the abnormal eosinophils. Prognosis is relatively favorable.

good = t(8;21), t(15;17), inv(16)
bad =11q23, FLT3

The Complete (but not obsessive) Hematopathology Guide
page 53
www.pathologystudent.com
AML wi t h t ( 15; 17)
All cases of AML with a t(15;17) have the same morphology: they fall into the category of acute
promyelocytic leukemia (or AML-M3 in the FAB classification). The marrow and blood are full of
promyelocytes (at least 20%, but usually more). These (malignant) promyelocytes look kind of like
benign promyelocytes, but they tend to have way more granules. One pathognomonic finding in
this type of AML is the presence of promyelocytes with TONS of Auer rods in them (like 40 or 50).
These cells are called faggot cells (faggot meaning bundle of sticks).
Patients with acute promyelocytic leukemia are at great risk for DIC (disseminated intravascular
coagulation) if given conventional treatment. The granules inside the promyelocytes have pro-
coagulant substances in them so if you bust open the cells (as normal chemotherapeutic agents
do), the patient is at great risk for clotting. This made treatment difficult before we understood
what was going on at the genetic level in these cells.
It turns out that the t(15;17) encodes for an abnormal retinoic acid receptor which blocks cell
differentiation. A drug called ATRA (all-trans retinoic acid) is now used for therapy in these patients.
It acts as a differentiating agent, allowing the screwed-up promyelocytes (with the screwed-up
retinoic acid receptor) to mature into neutrophils (and then die normally). Then, with most of the
dangerous granule-containing promyelocytes gone, you can treat the patient with cell-busting
chemotherapy. Prognosis is now favorable in these cases.

AML wi t h 11q23 abnormal i t i es
This type of AML usually has a monocytic component (most cases resemble either AML-M4 or
AML-M5 in the FAB classification). Two groups of patients are more likely to get this type of AML:
infants and patients who have been treated in the past with chemotherapy (for another tumor).
Unfortunately, in this type of AML, prognosis is poor (even more poor than the regularly poor
prognosis of AML).

AML wi th FLT3 mutati on
This type of AML has a mutation of FLT3 (duh), which is short for FMS-like tyrosine kinase-3. This
mutation is present in almost a third of cases of AML (wow!), making it the most common genetic
abnormality in AML. Most of these cases have a monocytic component (meaning they would have
been called AML-M4 or M5 using the FAB classification). Patients with AMLs with this mutation
have an increased relapse rate and shortened survival; their overall prognosis is poor.
Faggot cel l i n APL

The Complete (but not obsessive) Hematopathology Guide
page 54
www.pathologystudent.com
AML wi th mul ti l i neage dyspl asi a
AMLs that fall into this category have two things going on:
1. At least 20% blasts in blood or bone marrow
2. Dysplasia in at least two myeloid cell lines, including some of the following:
Dysgranulopoiesis (neutrophils with hypogranular cytoplasm,
hyposegmented nuclei, or bizarrely segmented nuclei)
Dyserythropoiesis (megaloblastic nuclei, karyorrhexis, or multinucleation
of erythroid precursors; ringed sideroblasts)
Dysmegakaryopoiesis (micromegakaryocytes, and normal-to-large sized
megakaryocytes with monolobed nuclei or multiple separated nuclei)
This type of AML often occurs in elderly patients; a frequent presentation is severe pancytopenia. It
may occur de novo or may evolve from a myelodysplastic syndrome (see page 59) or a
myeloproliferative disorder (see page 65). Patients often have chromosome abnormalities
(especially of chromosomes 5 and/or 7). It is hard to get patients with this type of AML into
remission, and the overall prognosis is poor.

AML, therapy-rel ated
This category includes both AML and myelodysplastic syndrome (MDS) that occurs as a result of
previous chemotherapy and/or radiation therapy. Most cases of therapy-related AML or MDS are
caused by one of two classes of chemotherapy drugs: alkylating agents and topoisomerase II
inhibitors.
AML/MDS caused by al kyl at i ng agent s
This type of AML usually occurs about 5-6 years after exposure to an alkylating agent, such as
busulfan, cyclophosphamide and ifosfamide. It usually starts out as a myelodysplastic syndrome,
and is often accompanied by bone marrow failure. Many patients die in the MDS phase, but in
some patients, the disease evolves into AML. Cytogenetic abnormalities are almost always present
(usually involving chromosomes 5 and 7).
AML/MDS caused by t opoi somerase I I i nhi bi t ors
This type of AML usually occurs about 2 years after exposure to topoisomerase II inhibitors, such
as epipodophyllotoxins (e.g., etoposide) or anthracyclines (e.g., doxorubicin). There is usually a
prominent monocytic component (most cases would be called M4 or M5A in the FAB
classification). Cytogenetic abnormalities are frequently present, and usually involve chromosome
11q23 and the MLL gene.

The Complete (but not obsessive) Hematopathology Guide
page 55
www.pathologystudent.com
AML, not otherwi se cl assi f i ed
Cases that dont fit into one of the other four WHO categories above are placed into this category.
This is a good place to discuss the FAB classification subtypes, many of which did not come into
our discussion above. Youre going to run into these names, so might as well learn a bit about
them.










AML-M0 ( AML, mi ni mal l y di f f erent i at ed)
This type of AML has a ton of myeloblasts in the blood and bone marrow (usually myeloblasts
comprise over 90% of the nucleated cells). However, as the name says, the myeloblasts show
minimal differentiation. This means that the malignant myeloblasts are stuck in such an early stage
of development that you almost cant even tell whether they are myeloid or lymphoid! They dont
have Auer rods, and they dont stain for MPO. So you need to use immunophenotyping to figure
out that the cells are myeloblasts.

AML-M1 ( AML wi t hout mat urat i on)
This type of AML also has a ton of myeloblasts in the blood and bone marrow (usually myeloblasts
comprise over 90% of the nucleated cells). However, in contrast to the myeloblasts in AML-M0,
these myeloblasts do show some differentiation. You might see a few Auer rods, and some of the
blasts will stain positively for MPO. As the name says, there is no maturation. Which means you
only see myeloblasts nothing more mature (like promyelocytes or myelocytes).
FAB Cl assi f i cati on of Acute Myel oi d Leukemi a
AML-M0 Acute myeloblastic leukemia, minimally differentiated
AML-M1 Acute myeloblastic leukemia without maturation
AML-M2 Acute myeloblastic leukemia with maturation
AML-M3 Acute promyelocytic leukemia
AML-M4 Acute myelomonocytic leukemia
AML-M5 Acute monocytic leukemia
AML-M6 Acute erythroid leukemia
AML-M7 Acute megakaryoblastic leukemia

M0 = # myeloblasts
M1 = # myeloblasts
(Hows it different from M0?)

The Complete (but not obsessive) Hematopathology Guide
page 56
www.pathologystudent.com

AML-M2 ( AML wi t h mat urat i on)
In this type of AML, there are enough myeloblasts to make the 20% cutoff, and theyre
differentiated (meaning you see signs of myeloid-ness, like Auer rods and MPO staining). However,
in contrast to AML-M1, there is maturation. In addition to the myeloblasts, youll see all the other
stages of neutrophil development (promyelocytes, myelocytes, metamyelocytes, and segmented
neutrophils). All of these cells are part of the same malignant clone its just that the malignant
cells are not stuck in the myeloblast stage; they are able to mature into segmented neutrophils.
As mentioned above (page 52), some cases of AML-M2 show a t(8;21). Patients with this type of
AML receive different chemotherapeutic agents, and have a better prognosis. You can often tell
when a case of AML-M2 has a t(8;21), even before you get the cytogenetic report back, because
of the peculiar way these cases look. The myelocytes often have a coalescence of specific
granules in the cytoplasm (which looks like a big pink cytoplasmic blob), as well as extra-long,
needle-like Auer rods. Weird.

AML-M3 ( Acut e promyel ocyt i c l eukemi a)
This type of AML is characterized by a proliferation of malignant promyelocytes. So to make the
diagnosis, you dont need to worry about myeloblasts; you just need to make sure that the
promyelocyte population comprises >20% of the nucleated cells. All (or virtually all) of these cases
of AML have a t(15;17) which, as discussed on page 53, results in the formation of an abnormal
retinoic acid receptor that interferes with the maturation of the cells (they get stuck in the
promyelocyte stage, and cant mature any further). Its cool that the initial treatment (all-trans
retinoic acid, or ATRA) targets this messed-up receptor, allowing the cells to differentiate. These
patients have a relatively good prognosis.

M4 ( Acut e myel omonocyt i c l eukemi a)
AML-M4 is kind of a dual-lineage leukemia: you see lots of myeloblasts, but also a lot of monocytic
cells (monoblasts and promonocytes). Combined together, all these must equal >20% of the
nucleated cells in the blood or marrow.
As mentioned above (page 52), some cases of AML-M4 have an inv(16). These cases usually have
an unusual morphology, in which there are lots of big eosinophils with dark granules. This variant is
called AML-M4 Eo, and it has a better prognosis than AML in general.

M2 = # myeloblasts +
maturing neutrophils
M3 = # promyelocytes
M4 = # promyelocytes

The Complete (but not obsessive) Hematopathology Guide
page 57
www.pathologystudent.com




M5 ( Acut e monocyt i c l eukemi a)
AML-M5 comes in two flavors: AML-M5A and AML-M5B. AML-M5A has lots of monoblasts, and
AML-M5B has lots of promonocytes. There may be a mixture of blasts (with some myeloblasts
thrown in) so the way its worded is: at least 20% of the cells must be blasts. If 80% or more of
those are monoblasts, its an M5A; if 80% or more are promonocytes, its an M5B.
AMLs with a monocytic component (that is, M4 or M5) have an increased incidence of
extramedullary involvement. Common sites are skin, central nervous system, gums, and testes.
Patients with these types of AML are often given different treatment (for example,
chemotherapeutic agents are injected into the CSF) to help prevent this type of spread.



AML-M6 ( acut e eryt hrobl ast i c l eukemi a)
This type of AML involves both the myeloid and erythroid lineages. There are lots of myeloblasts (at
least 20%) and also lots of erythroid precursors (usually these are greater than 50% of all nucleated
cells). The erythroid precursors may be giant and bizarre, with bilobed nuclei and multinucleation.




AML-M7 ( acut e megakaryobl ast i c l eukemi a)
This one is just what the name says: an AML that involves the megakaryocytic lineage. The blood
and bone marrow show at least 20% megakaryoblasts. Youd think these would look weird
(because megakaryocytes look so weird) but they often just look like bland, run-of-the mill blasts.
There arent any cytochemical stains that are specific for megakaryocytes or megakaryoblasts so
usually you need to resort to flow cytometry. Cells of the megakaryocytic lineage usually express
CD41 and/or CD61.

M5B = # promonocytes
M5A = # monoblasts
M6 = # erythroblasts + myeloblasts
M7 = # megakaryoblasts
AML-M5B

The Complete (but not obsessive) Hematopathology Guide
page 58
www.pathologystudent.com

Treatment and Prognosi s
Patients with AML generally get a big dose of chemotherapy to start with. It differs depending on
the type of AML (patients with acute promyelocytic leukemia, for example, receive ATRA, as
mentioned above). About two-thirds of patients enter a complete remission (meaning theres no
evidence of disease) after chemotherapy. Unfortunately, this remission is often short-lived.
Patients who have a bone marrow match, and who are able to tolerate transplant (meaning they
are relatively young and relatively healthy) are usually taken to bone marrow transplantation. This is
really the only chance for a cure for AML at this point but the procedure is not without risk
(particularly infection).
Even with bone marrow transplantation, prognosis is dismal. Overall, about 80% of patients die
within 3 years. The prognosis varies inversely with age (older patients do worse). Cytogenetics are
important in determining prognosis. Patients with t(8.21), inv(16) and t(15;17) survive longer;
patients with FLT-3 mutations or 11q23 abnormalities have a worse prognosis.


The Complete (but not obsessive) Hematopathology Guide
page 59
www.pathologystudent.com

Myelodysplastic Syndromes (MDS)
The myelodysplastic syndromes are a group of disorders characterized by dysplastic changes in
myeloid cells, with or without an increase in blasts. MDS occurs more frequently in older patients
(like around 60-70). It may be asymptomatic, or it may present with symptoms of bone marrow
failure (weakness, infections, bleeding). Many but not all cases evolve into acute leukemia.
Cl assi f i cati on
There are currently 6 types of MDS, and though we wont get into the particular requirements for
diagnosis (thats more for hematopathology boards), you can tell from the name what each one
involves: refractory anemia (RA), refractory anemia with ringed sideroblasts (RARS), refractory
anemia with excess of blasts (RAEB), chronic myelomonocytic leukemia (CMML), chronic
myelomonocytic leukemia in transformation (CMML-T), myelodysplastic syndrome, unclassified
(MDS-U).
Morphol ogy
As mentioned above, myelodysplastic syndromes are characterized by dysplastic changes in
myeloid cells. These changes look different in each cell line:
Red cell dysplasia - nuclear lobulation and fragmentation.
Neutrophil dysplasia - hypogranulation, hyposegmentation
Megakaryocytic dysplasia - small megakaryocytes with non-lobulated nuclei
In addition to the dysplastic changes, you may see increased numbers of blasts. However,
remember that if you see > 20% blasts, you must classify the case as acute leukemia!).
One other morphologic finding of note: most patients with MDS present with a macrocytic anemia.
Remember the other macrocytic anemias we talked about? If not, see pages 17 and 36. If you
have a patient with a macrocytosis (MCV >100), particularly if your patient is an older patient, MDS
is something you need to consider.
Treatment and Prognosi s
Some types of MDS (like refractory anemia) tend to be lower-grade, meaning that they rarely
evolve into acute leukemia. These are treated supportively (with transfusions if necessary but no
big time chemotherapy). The course in these cases is long, and the prognosis is relatively good.
Other types of MDS (like refractory anemia with excess of blasts) tend to be more aggressive, with
rapid evolution to AML in about one third of the patients. These cases are treated with aggressive
anti-leukemic therapy.
MDS = dysplasia # blasts

The Complete (but not obsessive) Hematopathology Guide
page 60
www.pathologystudent.com
Acute Lymphoblastic Leukemia

Acute lymphoblastic leukemia (ALL) is a malignant proliferation of lymphoblasts in the bone marrow
and blood. In ALL, theres no 20% blast cutoff for diagnosis like there is in AML. Usually, theres
way more than that often most of the nucleated cells in the bone marrow and blood are blasts.
ALL is the most common type of leukemia in children (about 80% of all childhood leukemias are
ALL). Most cases occur in patients under 15 years of age, but a small number of cases occur in
adults. The prognosis is relatively good for children, but poor for adults.

Cl assi f i cati on
ALL is classified according to immunophenotype. Its pretty straightforward:



TdT T-cell Ag B-cell Ag
T-cel l ALL + + -
Precursor-B ALL + - +
Burki t t l eukemi a - - +


The two big divisions are between T-lineage and B-lineage leukemias. The T-lineage leukemias are
positive for TdT (terminal deoxynucleotidal transferase, an enzyme present in lymphoblasts) and for
T-cell antigens (like CD3).
The B-lineage leukemias are divided into precursor-B ALL (which is positive for TdT and B-cell
antigens (like CD21), and Burkitt leukemia (or B-cell ALL) (which is positive for B-cell antigens, but
negative for TdT). Lets look at the different types.

T-cel l ALL
T-cell ALL has markers that are really easy to remember (TdT positive, T-cell-antigen positive). It
accounts for about 15% of all cases of ALL, and its more common in the teenage years (perhaps
because thats the age at which the thymus reaches its maximal size). It often presents with a
mediastinal mass and a very high white blood count. Prognosis has traditionally been poor, but
with intensive chemotherapy, prognosis now approaches that of precursor-B ALL. It is the same
disease as T-lymphoblastic lymphoma.
ALL is the most common
leukemia in children.
T-cell ALL =
T-lymphoblastic lymphoma

The Complete (but not obsessive) Hematopathology Guide
page 61
www.pathologystudent.com

Precursor-B ALL
Precursor-B ALL also has markers that are easy to remember (TdT positive, B-cell-antigen
positive). Its by far the most common type of ALL, accounting for about 80% of all cases. Heres
another leukemia/lymphoma pairing: B-cell precursor ALL is the same thing as B-lymphoblastic
lymphoma, which well discuss later. The prognosis is pretty good, especially in children.

Burki tt l eukemi a
Burkitt leukemia (or, as it was known before the latest classification change, B-cell ALL) has a
weird immunophenotype: its TdT negative! Weird, considering its a leukemia composed of
lymphoblasts, and TdT is a lymphoblast marker. Whatever. The morphology is unusual too: the
lymphoma cells are big cells with deep blue cytoplasm and sharply punched-out vacuoles. Most
cases have a translocation between chromosomes 8 and 14, involving the c-myc gene on
chromosome 8 and the heavy chain gene on chromosome 14. Sometimes, instead of
chromosome 14, chromosomes 2 or 22 are involved. The kappa light chain gene is on
chromosome 2 and the lambda light chain gene is on chromosome 22 (its easy to remember: the
earlier letter goes with the smaller number).
Burkitt leukemia accounts for about 1% of all cases of ALL; the prototypic patient is an older child
with a fast-growing abdominal mass. Burkitt leukemia is the same disease as Burkitt lymphoma
(the official name is: Burkitt leukemia/lymphoma). Whatever you want to call it, its an extremely
fast-growing tumor. Which means it has this paradox of being aggressive but also relatively
sensitive to chemotherapy (traditional chemotherapy hits dividing cells so if a tumor has a lot of
dividing cells, its more sensitive to chemotherapy).

How to make a di agnosi s
Morphol ogy
In ALL, there are lots of blasts in the blood and bone marrow. Remember in AML how you had to
count the blasts and make sure that they numbered at least 20% of the nucleated cells in the
blood or bone marrow? Well, as mentioned above, that doesnt hold for ALL. Theres no cutoff.
You do count the cells, because its important to know how heavy the tumor burden is but the
presence of even a small number of lymphoblasts is enough to make the diagnosis. In many
cases, the marrow and blood are totally stuffed full of lymphoblasts, and counting is kind of a moot
point.
Precursor-B ALL =
B-lymphoblastic lymphoma
Burkitt leukemia =
Burkitt lymphoma
Burki t t l eukemi a

The Complete (but not obsessive) Hematopathology Guide
page 62
www.pathologystudent.com



Lymphoblast morphology is not very exciting. Some lymphoblasts are large, with big nuclei and
fine chromatin; others are smaller with more condensed chromatin. In ALL, it is often hard to tell
whether the blasts are lymphoid or myeloid just by looking at them under the microscope, because
usually lymphoblasts are pretty non-descript. Unlike the myeloblasts in AML, which may have Auer
rods in them, the lymphoblasts in ALL do not have any definitive distinguishing characteristics.
One soft clue that youre dealing with a lymphoid (as opposed to myeloid) process is the presence
of ghost or basket cells. These are remnants of lymphoblasts that have been busted open
during the preparation of the slide. Lymphoid cells are more fragile than myeloid cells, and often
they will burst as the blood (or bone marrow) is spread across the slide. These ghost cells can be
seen in any lymphoid process (even a benign lymphocytosis), so they dont help in differentiating
between a benign and a malignant process; but they do help a bit in differentiating between
myeloid and lymphoid. The presence of these cells is a pretty soft sign though. Youd want to do
markers to make a definitive diagnosis.

Cyt ochemi cal st ai ns
There arent any cytochemical stains for lymphoblasts. You can do an MPO or NSE to look for a
myeloid process. If the MPO and/or NSE are positive, you know its AML. But remember, the
blasts in AML-M0 and AML-M7 are generally negative for MPO and NSE! So even with a negative
MPO and NSE, it still might be AML.

I mmunophenot ypi ng
Immunophenotyping is a helpful tool for diagnosing ALL. You cant tell what the blasts are by
looking at them under the microscope (except for Burkitt leukemia cells, which look bizarre), and
cytochemical staining may be suggestive (but not definitive). So you really need to see what
markers are on the surface of the cells. You need this information first of all to tell if its an ALL (as
opposed to an AML), and second to classify the ALL. Usually, the flow lab will have an ALL panel
that they do for every potential case of ALL. It includes B-cell markers (like CD21), T-cell markers
(like CD3), TdT (produced in the nucleus) and immunoglobulin (this can be either in the cytoplasm
or on the cell surface). Some cases of ALL will also show aberrant expression of myeloid markers;
but its usually easy to tell that its ALL because most of the markers are lymphoid.


The Complete (but not obsessive) Hematopathology Guide
page 63
www.pathologystudent.com

Cyt ogenet i cs/Mol ecul ar St udi es
Two cytogenetic abnormalities are worth noting here: t(9;22) and t(8;14). Some cases of precursor
B ALL have the Philadelphia chromosome (the mutated chromosome 22 in the t(9;22)
translocation). This is strange, because the Philadelphia chromosome is a characteristic finding in
chronic myeloid leukemia. This translocation is more likely to show up in cases of ALL occurring in
adults, and it is a poor prognostic indicator.
The t(8;14) translocation is characteristic of Burkitt leukemia. As mentioned above, the
translocation involves the c-myc gene on chromosome 8 and the heavy chain locus on
chromosome 14. The c-myc gene product activates transcription (in other words, it makes the cell
grow). If you put the c-myc gene next to the heavy chain gene (which is always being transcribed),
then youve set up the cell to grow like crazy. Sometimes, instead of winding up next to the heavy
chain locus on chromosome 14, c-myc moves next to one of the light chain genes (the kappa
gene is on 2 and the lambda gene is on 22). The same thing happens in these t(2;8) and t(8:22)
translocations: c-myc gets transcribed like crazy, making the cell grow like crazy.

Treatment and Prognosi s
There are several factors that you can look at to help determine prognosis in a patient with ALL:
Immunophenotype (best prognosis: precursor B ALL)
Age (best prognosis: patients between 1 and 10 years of age)
WBC (best prognosis: less than 10 x 10
9
/L)
Cytogenetics (best prognosis: normal or hyperdiploid tumor cells)
Treatment in ALL is similar to that in AML, and involves chemotherapy with or without bone
marrow transplantation. Fortunately, the prognosis is relatively good for most children with ALL;
with aggressive chemotherapy, about 80% are cured. Adults do not fare as well; only about 40%
of adults with ALL are cured of their disease.



ALL prognosis in kids is good.

The Complete (but not obsessive) Hematopathology Guide
page 64
www.pathologystudent.com

Chronic Leukemias
Chronic leukemias are very different from acute leukemias. Chronic leukemias are for the most part
diseases of older adults (acute leukemias occur in both children and adults). They appear in an
insidious fashion and have a relatively good prognosis (as opposed to acute leukemias, which
have a stormy onset and poor prognosis). In addition, chronic leukemias are composed of fairly
mature-appearing hematopoietic cells (as opposed to acute leukemias, which are composed of
blasts).
There are two kinds of chronic leukemias: myeloid and lymphoid. Instead of being reasonable, and
calling them chronic myeloid leukemias and chronic lymphoid leukemias, the powers that be
dubbed the two divisions chronic myeloproliferative disorders and chronic lymphoproliferative
disorders. These names are not so great, in my opinion, since these are not just disorders
they are real leukemias! But no one asked me.

Pathophysi ol ogy
The chronic leukemias are malignant, monoclonal proliferations of mostly mature myeloid or
lymphoid cells in the bone marrow (and blood). These leukemias progress more slowly than acute
leukemias. So early on, the marrow is involved but not totally replaced by malignant cells. Still,
it is hard for the normal white cells to function properly. The lymphoid cells, in particular, have a
hard time making normal immunoglobulin in certain chronic lymphoproliferative disorders. One of
the major causes of mortality in these patients is infection. As the chronic leukemias evolve, more
and more of the marrow is replaced by tumor, and eventually there is little room for normal white
cells to grow.

Cl i ni cal Features
Chronic leukemias present in over a period of weeks or months. Patients might have splenomegaly
(which shows up as a dragging sensation or fullness in the left upper quadrant of the abdomen),
lymphadenopathy, or a general feeling of malaise and fatigue. Some patients are asymptomatic at
diagnosis, and the disease is picked up on a routine blood smear or CBC. Likewise, the clinical
course is different in chronic leukemia. In many cases of chronic leukemia, patients can live for
years without treatment at all.

The Complete (but not obsessive) Hematopathology Guide
page 65
www.pathologystudent.com


Chronic Myeloproliferative Disorders
The chronic myeloproliferative disorders are malignant clonal proliferations of a pluripotent stem
cell that lead to excessive proliferation of myeloid cells in the blood and bone marrow. What that
means in plain English is that a stem cell somewhere way back (before its even committed to the
neutrophil line, or red cell line) goes bad and starts proliferating like crazy so you wind up with a
marrow packed with cells from all the myeloid lineages (the official name is panhyperplasia).
Usually, one particular myeloid lineage predominates in this growth fest so youll see a ton of all
the myeloid cells, but the majority are neutrophils, or red cells, or megakaryocytes. So the chronic
myeloproliferative disorders have been divided into four types according to what is proliferating
most:
Chronic myeloid leukemia (tons of neutrophils and precursors)
Polycythemia vera (tons of red cells and precursors)
Essential thrombocythemia (tons of platelets and megakaryocytes)
Chronic myelofibrosis (tons of everythingthen nothing! See below.)
Well consider each of these separately because they are very different clinically and
morphologically. But they do have some common features: all of them have a high white count
with a left shift, a hypercellular marrow, and splenomegaly.

Chroni c myel oi d l eukemi a
Chronic myeloid leukemia (CML) is a chronic myeloproliferative disorder characterized by a marked
proliferation of neutrophils (and precursors) in the bone marrow and blood. All cases have a t(9;22),
also known as the Philadelphia chromosome (its the 22 thats officially the Philadelphia
chromosome).

Cl i ni cal Feat ures
CML frequently occurs in patients who are around 40 or 50. It does not occur in children (though
there is a separate disease similar to CML, called juvenile CML, that does occur in kids). Usually,
the onset is slow, with a long asymptomatic period, followed by fevers, fatigue, night sweats and
abdominal fullness. On physical exam, patients usually have an enlarged spleen. Hepatomegaly
and lymphadenopathy may also be present.
CML, PV, ET, and
Chronic myeloproliferative disorders:
chronic myelofibrosis
CML has a t(9;22).

The Complete (but not obsessive) Hematopathology Guide
page 66
www.pathologystudent.com

There are three clinical stages, or phases, of CML: chronic phase, accelerated phase and blast
crisis. Patients generally present in chronic phase and then progress to one or both of the other
phases.
Chroni c phase
High but stable number of neutrophils and precursors.
Stable hemoglobin and platelet count.
Easily controlled by therapy.
With traditional treatment (not imatinib, see below), usually lasts 3-4 years; is then followed
by accelerated phase and/or blast crisis.
Accel erat ed phase
Characterized by a change in the patient's previously stable state.
Usually see increasing leukocytosis, decreasing hemoglobin and platelet count.
May terminate in this stage, or may progress to blast crisis.
Usually fatal within several months.
Bl ast cri si s
Characterized by a marked increase in blasts (myeloblasts or lymphoblasts).
Usually fatal within a few weeks or months.

Morphol ogy
Blood
The blood smear shows a marked neutrophilia with a left shift. The left shift is a little weird in that it
is not evenly distributed between all the neutrophil stages. There are tons of neutrophils at all
stages of development, but there are relatively more myelocytes and segmented neutrophils (and
relatively less of the other stages). There are a few myeloblasts around (which you dont see in
normal blood, of course) but they dont number more than 2 or 3%.
Heres an interesting thing: patients with CML almost always have a basophilia. Thats actually one
of the first things that happens in the development of the disease! There are few if any other
reasons for a basophilia. So if you see this in a patient, even if they dont have the typical findings
of CML (big white count with lots of neutrophils and precursors), you should rule out CML!
The platelet count may be increased (because of all the megakaryocytes around in the bone
marrow).
Chronic myeloid leukemia:
marked neutrophilia,
left shift, and basophilia
CML: bl ood

The Complete (but not obsessive) Hematopathology Guide
page 67
www.pathologystudent.com

Bone marrow
The bone marrow is hypercellular, with a pan-myeloid hyperplasia (all the myeloid cells are
increased neutrophils and precursors, red cell precursors, megakaryocytes). However, if you look
closely, youll see that the neutrophils and precursors make up the bulk of the cells. Later in the
course of the disease, the marrow may become fibrotic. You can detect this using a reticulin stain.
This is not a good sign.

Pat hophysi ol ogy
All cases of CML have a translocation between chromosomes 9 and 22, resulting in whats
commonly known as the Philadelphia chromosome (Ph). This designation refers to the new
chromosome 22 that results from the translocation. Nobody talks about poor chromosome 9. The
translocation places the c-abl proto-oncogene on chromosome 9 next to the bcr gene on
chromosome 22. A new, fusion gene is created: the bcr-abl gene. The bcr-abl gene encodes a
protein called p210, which increases tyrosine kinase activity and disrupts the cell cycle.
Heres a weird fact: the Philadelphia chromosome is found not only in the myeloid cells, but also in
some B lymphocytes! Thats weird, considering that this is a myeloid lesion with no apparent
changes in the lymphoid cells. This probably means that the initial bad cell (the one that became
malignant) was a very early stem cell, one that hadnt even committed itself to myeloid or lymphoid
lineage yet, and the Philadelphia chromosome is present in all the descendents of that cell. Further
supporting this idea is the fact that when patients enter blast crisis, the blasts can be lymphoid!

Treat ment and Prognosi s
In the old days, CML was treated with myelosuppressive agents like hydroxyurea, and then if the
patient had a match and could tolerate it, allogeneic bone marrow transplant was performed. That
was the only hope for a cure.
Recently, a new drug called imatinib (or Gleevec) was developed that targets the messed-up
tyrosine kinase receptor activity in CML. It has been like a miracle for many patients even patients
in the later stages of the disease. In fact, we dont even know what the typical prognosis of CML is
anymore, because these patients are still living with the disease. This drug has turned CML into a
chronic but treatable disease, like diabetes, for many patients. Its one of the happiest leukemia
research stories ever.

CML: bone marrow

The Complete (but not obsessive) Hematopathology Guide
page 68
www.pathologystudent.com


Chroni c myel of i brosi s
Chronic myelofibrosis, also called idiopathic myelofibrosis or agnogenic myeloid metaplasia, is a
chronic myeloproliferative disorder characterized by panmyelosis, bone marrow fibrosis, and
extramedullary hematopoiesis. In real words: the bone marrow at first is markedly hypercellular, but
over time, it becomes fibrotic, and the hematopoietic cells go elsewhere (most often, to the spleen)
to try to make a home.

Cl i ni cal Feat ures
Like the other chronic leukemias, chronic myelofibrosis is a disease of older adults. The peak age is
in the late 50s. The disease presents over a relatively long period of time, with symptoms of
splenomegaly (left upper quadrant pain and fullness, epigastric pressure) and anemia (weakness,
fatigue, and palpitations). A small number of patients are asymptomatic at diagnosis. Physical
examination shows massive splenomegaly in most patients, as well as signs of anemia (pallor,
tachycardia).

Morphol ogy
Blood
The blood smear shows a leukoerythroblastosis (remember this term from benign leukocytoses? If
not, see page 41). There are lots of teardrop red cells in the blood due to the tight spaces (fibrotic
marrow, big spleen) the red cells have to squeeze through. The platelets are often weird looking
(large and hypogranular).
Bone marrow
Early on, the marrow is hypercellular, with a pan-myeloid hyperplasia. Megakaryocytes, in
particular, are markedly increased in number. Later on, the marrow becomes fibrotic, and in the
end stages of the disease, it is entirely replaced by fibrotic tissue, with very few remaining
hematopoietic cells.

Pat hophysi ol ogy
The cause of the fibrosis is still not completely worked out. The fibroblasts are benign so why are
they so active? Its probably a result of megakaryocyte stimulation. Megakaryocytes are known to
release cytokines that stimulate fibrosis and in chronic myelofibrosis, there are tons of
megakaryocytes around. So this seems like a plausible explanation.
Chronic myelofibrosis:
marrow fibrosis,
extramedullary hematopoiesis
and teardrop red cells
Chroni c myel of i brosi s: bl ood
Chroni c myel of i brosi s: bone marrow

The Complete (but not obsessive) Hematopathology Guide
page 69
www.pathologystudent.com


Treat ment and Prognosi s
Chronic myelofibrosis has a relatively long course (mean survival is 5 years). Treatment usually
consists of supportive measures (like red cell transfusions) and myelosuppressive therapy (like
hydroxyurea) if the patient can tolerate it. The cause of death in patients with chronic myelofibrosis
is usually marrow failure. A small number of patients undergo leukemic transformation (meaning
that they develop an acute leukemia like the blast crisis phase we talked about in CML).
Interestingly, the acute leukemia can be either myeloid or lymphoid!

Pol ycythemi a Vera
Polycythemia vera (PV) is a chronic myeloproliferative disorder characterized by panmyelosis, with
an erythroid predominance. In real words: the marrow is stuffed with myeloid cells, and most of
them are red cell precursors. The blood shows a markedly increased red cell count.
Polycythemia" just means an increase in red blood cell mass. It may be:
Primary (polycythemia vera or true polycythemia): increase in red blood cells caused by an
intrinsic abnormality of myeloid cells (no # in erythropoietin).
Secondary: increase in red blood cells caused by # secretion of erythropoietin, which may
be appropriate (e.g., high-altitude living) or inappropriate (e.g., a paraneoplastic syndrome
related to a solid tumor).
To distinguish between primary and secondary polycythemia, and to differentiate PV from other
chronic myeloproliferative disorders, a polycythemia vera study group came up with the following
Polycythemia Vera Study Group Criteria (how creative). To diagnose PV, you need either (1) A1,
A2, and A3, or (2) A1, A2, and any two from B:
A ( maj or) cri t eri a
A1 Increased RBC mass
A2 Normal O
2
saturation in blood
A3 Splenomegaly
B ( mi nor) cri t eri a
B1 Thrombocytosis
B2 High WBC without infection
B3 Increased leukocyte alkaline phosphatase (see box at right) without infection
B4 Increased serum B
12
level

Things to make you look smart
Q. What i s l eukocyt e al kal i ne
phosphatase ( LAP) ?
A. LAP is an enzyme present in
normal neutrophils. It is strangely
absent in the neutrophils in CML so
it was used in the olden days (before
cytogenetics) to distinguish between a
benign neutrophilia and CML. LAP is
expressed at normal levels in the
neutrophils in PV. Why is that? Who
knows! But its important because if
you have a lot of neutrophils around,
and you think its a chronic
myeloproliferative disorder, you could
do an LAP. If it was low or zero, that
would be a pretty good indicator of
CML. If it was increased, it could be
PV (youd still have to rule out
infection though). Nowadays, we use
other means to diagnose these
disorders. But youll still hear people
(and books) talk about the LAP. So
now you know.
Polycythemia vera:
tons of red cells

The Complete (but not obsessive) Hematopathology Guide
page 70
www.pathologystudent.com
Cl i ni cal Feat ures
Polycythemia vera, like the other chronic leukemias, is a disease of older adults; the mean age at
diagnosis is 60. Symptoms and signs are related to the massive increase in red cell mass. Patients
may have headaches, weakness, pruritis and dizziness (from increased blood volume); they may
also have symptoms of vascular stasis, thrombosis or infarction (from increased blood viscosity).
Physical examination may show hepatosplenomegaly, and something called plethora, which
means ruddiness or redness of the head and neck.
Morphol ogy
Blood
For most of the course of the disease, the red cell count is markedly increased. So are the white
cell count and the platelet count. Towards the end stages of the disease, though, the marrow can
become pooped out (the official name for this is spent phase) and quit making red cells. Then the
patients red cell count goes down, and the patient may even become anemic.
Bone marrow
The marrow is hypercellular, with a pan-myeloid hyperplasia. Red cells make up the bulk of the
myeloid cells in the marrow. Towards the end of the disease, however, the marrow may become
fibrotic, and red cell production may decrease.
Pat hophysi ol ogy
Recently, a unique genetic abnormality was found to be present in virtually all patients with
polycythemia vera. There is a normal signaling pathway present in all kinds of organisms, from
slime molds to humans, called the JAK-STAT (Janus kinase-signal transducer and activator of
transcription) pathway. Its a pretty cool pathway because it transmits signals from outside of the
cells (like growth hormone signals) to the nucleus of the cell without the need for second
messengers (which most other receptors need to use). It turns out that myeloid growth and
development is mediated, in part, by this pathway. Patients with polycythemia vera have a
mutation in the JAK part of this pathway (specifically, in the JAK-2 gene), which makes the JAK
think its getting signals when its not. So cells with this mutation are constantly getting signals to
grow! The mutation has been found in virtually all cases of polycythemia vera (making it a great tool
for differentiating primary from secondary polycythemia), and in a significant number of patients
with chronic myelofibrosis and essential thrombocythemia too.
Treat ment and Prognosi s
Treatment usually involves phlebotomy, with or without myelosuppressive drugs. Survival is long
(average 9 - 14 years). Dangers include thrombosis, hemorrhage, and transformation into acute
leukemia.
Virtually all cases of
JAK-2 mutation
polycythemia vera have a

The Complete (but not obsessive) Hematopathology Guide
page 71
www.pathologystudent.com
Essenti al Thrombocythemi a
Essential thrombocythemia (ET) is a chronic myeloproliferative disorder characterized by
panmyelosis, with a megakaryocytic predominance. Meaning: the marrow is stuffed full of myeloid
cells, and the predominating cell is the megakaryocyte. The blood shows a markedly increased
platelet count.
The diagnosis of ET is basically one of exclusion. You have to rule out benign causes of
thrombocytosis and all of the other chronic myeloproliferative disorders. Here are the criteria:
Platelet count must be >600 x 10
9
/L (normal = 150 450).
Hgb must be <13 g/dL or RBC mass must be normal (excludes PV).
Philadelphia chromosome must be absent (excludes CML).
Marrow must lack fibrosis (excludes chronic myelofibrosis).
All other causes of thrombocytosis (e.g., iron deficiency anemia, cancer) must be excluded.

Cl i ni cal Feat ures
ET usually occurs in patients over 50, but occasionally it occurs in young women. Symptoms are
those related to thrombotic phenomena (like myocardial infarction, stroke, and deep venous
thrombosis). Some patients also have excessive bleeding, which you wouldnt expect, if you have
all those platelets around. The cause is a secondary (or acquired) von Willebrand disease! Weird.
Physical examination may reveal mild splenomegaly, pallor and tachycardia (if the patient is
anemic), and purpura and ecchymoses (if the patient has developed a secondary von Willebrand
disease).

Morphol ogy
Blood
The blood shows a ton of platelets - usually the count is over a million. The platelets usually look
abnormal; they are often large and/or hypogranular.
Bone marrow
The bone marrow shows normal to increased cellularity, and tons of megakaryocytes, which are
often described as being back-to-back.

Treat ment and Prognosi s
The main aim of treatment is to reduce the chances of hemorrhage or thrombosis. Usually platelet
pheresis or myelosuppressive drugs are used to lower the platelet count. Aspirin is also used to
decrease the risk of clotting. Most patients survive at least 5-8 years. Death is usually related to
hemorrhage or thrombosis, but a small number of cases undergo leukemic transformation.
Essential thrombocythemia:
tons of platelets
ET: bl ood
ET: bone marrow

The Complete (but not obsessive) Hematopathology Guide
page 72
www.pathologystudent.com
Chronic Lymphoproliferative Disorders
The chronic lymphoproliferative disorders are malignant clonal proliferations of relatively mature
lymphoid cells. These disorders are not stem cell malignancies (like chronic myeloproliferative
disorders), but malignancies that began in a lymphoid cell at some particular stage of development.
Its like the cell just gets stuck at this particular stage and cant mature any further.
Well be talking about four chronic lymphoproliferative disorders:
Chronic lymphocytic leukemia
Hairy cell leukemia
Prolymphocytic leukemia
Large granulated lymphocyte leukemia
Well consider each of these separately because they are very different clinically, morphologically
and immunophenotypically.

Chroni c l ymphocyti c l eukemi a
Chronic lymphocytic leukemia (CLL) is a relatively common chronic lymphoproliferative disorder
characterized by a sustained proliferation of mature-appearing lymphocytes. Basically, the marrow
and blood are full of mature-appearing (but malignant) lymphocytes. Most cases of CLL are of B-
cell type, but a very small minority are of T-cell type (nobody talks much about these).

Cl i ni cal Feat ures
CLL is a disease that only occurs in adults (never in kids). Its not too often in pathology that you
see always/never statements like that! In fact, if you see always or never on a pathology test, be
very wary of picking that answer. But I digress. Patients with CLL are usually older (>40). Thats
important to know, because this disease looks for all the world like a benign, mature lymphocytosis
(like one youd see in infectious lymphocytosis). The patients age can be a very important piece of
information when youre trying to make a diagnosis.
Patients with CLL are often asymptomatic for quite some time. The disease may be picked up
incidentally on a CBC done for some other reason. When symptoms do occur, they are related to
organ infiltration (lymphadenopathy, hepatosplenomegaly) or to infection (some patients with CLL
have a hypogammaglobulinemia, which makes it difficult to fight off infections).
Remember how we talked about lymphoma/leukemia pairs? We already discussed a few:
precursor T-lymphoblastic leukemia/lymphoma, precursor B-lymphoblastic leukemia/lymphoma,
CLL = SLL

The Complete (but not obsessive) Hematopathology Guide
page 73
www.pathologystudent.com
and Burkitt leukemia/lymphoma. Well, heres another one: regular old B-cell CLL is the same thing
as a lymphoma called small lymphocytic lymphoma (they are now lumped together under the name
B-cell CLL/SLL). Its actually kind of nice for students, because it cuts down the number of
diseases to memorize! Learn about CLL and you already know about SLL.

Morphol ogy
Blood
The blood smear shows a proliferation of mature-appearing lymphocytes (small cells, with
clumped/smudged chromatin). The white count may be massively elevated, or just slightly above
normal.
Bone marrow
Proliferation of mature lymphocytes. The more marrow involved, the worse the prognosis.

I mmunophenot ype
Immunophenotyping is an important part of diagnosing any of these chronic lymphoproliferative
lesions. Some of them can look quite similar, or can look like other lymphoid malignancies and
the only way you can really tell them apart is by doing immunophenotyping.
Regular old CLL (not the rare T-cell type) has a weird immunophenotype that makes it easy to
diagnose. It is:
negative for TdT (thats whats youd expect; TdT is an enzyme found in lymphoblasts)
positive for the B-cell antigens CD19, CD20, CD23 (yeah, okay, its a B-cell neoplasm)
positive for monoclonal surface Ig (sounds good, B cells make immunoglobulin)
positive for the T-cell antigen CD5 (whoa, wait a minute!)
So the CLL cell looks just like a maturing B cell, but it has aberrant expression of CD5 (a marker
you normally associate with T cells). It turns out that for a very short period of time during
maturation, B cells express CD5 (but then lose it again). So maybe thats where the CD5
expression in CLL comes from.

Pat hophysi ol ogy
A small number of cases show rearrangement of bcl-2, a protein involved in apoptosis. This
rearrangement leads to over-expression of bcl-2, prevention of apoptosis, and cell immortality.
Which kinda fits whats going on clinically in patients with CLL the cells dont grow explosively
(like the cells in Burkitt lymphoma), they just sort of dont die off. The most common chromosomal
abnormalities is trisomy 12 (which confers a bad prognosis).
CLL is a B-cell neoplasm
with aberrant expression of CD5.
CLL: bl ood

The Complete (but not obsessive) Hematopathology Guide
page 74
www.pathologystudent.com
Treat ment and Prognosi s
Patients with CLL are not really treated until they become symptomatic. The treatment, at least
currently, is often worse than just existing with the disease (and it doesnt prolong survival). When
treatment is given, it usually consists of corticosteroids and chemotherapy. Bone marrow
transplant is not useful.
There are a few staging systems that help predict survival in CLL. The Rai and Binet systems are
the most common. Both use several variables (like white count, lymphadenopathy,
hepatosplenomegaly, anemia, and thrombocytopenia) to predict median survival. The most
important adverse variables appear to be anemia and thrombocytopenia, surprisingly (youd expect
it to be the white count!). The amount and pattern of bone marrow involvement is also helpful in
determining prognosis; the more marrow involvement there is, and the more diffuse (as opposed to
patchy) the pattern is, the worse the prognosis is.
The prognosis varies quite a bit from patient to patient (some patients live many years, others have
rapid course), but overall, the prognosis is pretty good (the mean is somewhere around 10 years).
If a patient dies of the disease, it is usually due to infection. Some patients develop an aggressive
large-cell lymphoma (this is called Richters transformation) which usually heralds a terminal
phase of the disease.

Hai ry Cel l Leukemi a
Hairy cell leukemia (HCL) is a rare chronic lymphoproliferative disorder of B-cell origin in which the
cells look hairy. The primary sites involved are blood, bone marrow, and spleen.

Cl i ni cal Feat ures
HCL usually affects older patients (40-60), and it has a weird male predominance (M:F 5:1).
Patients usually have a huge spleen. Funny, though they usually dont have any
lymphadenopathy (usually, if you have a lymphoid neoplasm with a big spleen, youll have
lymphadenopathy too).

Morphol ogy
Blood
Patients with HCL are often pancytopenic. (Thats weird usually patients with leukemia have a
high white count!) Well see why that is in a minute. The few leukemic cells that are present have a
shaggy appearance the cytoplasm appears to have kind of a messy, blurry border. Thats
because the cells actually have very fine cytoplasmic projections. If you look under an electron
microscope, they really look like hairs! Another unusual feature: monocytopenia is always present!
Even in cases without a pancytopenia. Why is that? Who knows.
HCL: pancytopenia
(or at least monocytopenia)

The Complete (but not obsessive) Hematopathology Guide
page 75
www.pathologystudent.com
Bone marrow
The bone marrow has an unusual appearance. Its hypercellular (meaning there arent many fat
spaces), but loosely structured (its said to have a "moth-eaten" appearance). In part, this is due to
the characteristics of the hairy cells: they have quite a bit of cytoplasm, and these hairy things too,
which tend to push the cells apart from each other. So when you look at a bunch of hairy cells, the
nuclei arent closely smushed together (like CLL/SLL cells would be), but spaced apart, imparting a
loose appearance at low power.
Another factor leading to the moth-eaten appearance is reticulin fibrosis. For some reason, the
hairy cells elicit a fibrotic response in the marrow, and you get little strands of reticulin surrounding
each cell, kinda like chicken wire. This makes it pretty difficult for the cells to get out, too, if you
think about it so thats why the blood and bone marrow aspirate are pancytopenic.

Cyt ochemi st ry
Heres one of the few times that cytochemical stains are of use in lymphoid neoplasms. Theres a
special stain called tartrate-resistant acid phosphatase (or TRAP) thats positive in hairy cells (but
not in other small lymphocytes). What you do is stain the slide with acid phosphatase (which stains
a whole bunch of things red, like neutrophils, monocytes, and platelets), and then wash it with
tartrate (which washes the red out of almost every cell except hairy cells).

I mmunophenot ype
Heres the immunophenotype for hairy cell leukemia:
positive for B-cell antigens (CD19, CD20)
positive for CD25 (a marker that you normally see on monocytes and T cells)
positive for CD11c (also on monocytes and T-cells)
negative for CD5 (the T-cell antigen expressed in CLL).

Treat ment and Prognosi s
In the olden days, the only treatment for HCL was splenectomy and observation (with a relatively
grave prognosis). Now, using new chemotherapeutic agents (2-chlorodeoxyadenosine,
pentostatin, or interferon-alpha), the 5-year survival rate is over 85%.

HCL: hai ry cel l ( top) and
normal l ymphocyt e ( bot t om)
Hai ry cel l s posi ti ve f or TRAP

The Complete (but not obsessive) Hematopathology Guide
page 76
www.pathologystudent.com

Prol ymphocyti c l eukemi a
Prolymphocytic leukemia (PL) is a rare chronic lymphoproliferative disorder in which the
predominant cell is the prolymphocyte. Most cases of PL are of B-cell type, but a minority are of T-
cell type.

Cl i ni cal Feat ures
PL generally occurs in older patients. It usually presents with systemic disease, including
splenomegaly. It is more aggressive than the other chronic lymphoproliferative disorders.

Morphol ogy
Blood
The blood smear shows a proliferation of lymphocytes, over half of which are prolymphocytes.
Prolymphocytes are cells that you normally dont see in the blood or bone marrow. Theyre large
cells with coarse chromatin and a single, prominent nucleolus.
Bone marrow
The bone marrow shows a proliferation of the same type of cells.

I mmunophenot ype
Cases of PL that are of B-cell lineage show the following immunophenotype:
positive for B-cell antigens (CD20, CD22)
usually negative for CD5 (but sometimes positive)
T-lineage PL shows the immunophenotype of a mature T lymphocyte:
positive for T-cell antigens (CD2, CD3, CD7)
negative for TdT
Many cases are CD4+/CD8-; a smaller number are CD4+/CD8+ or CD4-/CD8+

Treat ment and Prognosi s
Both T-cell and B-cell PL are aggressive disorders. The B-cell type is somewhat responsive to
standard chemotherapy, but the survival is still short. The T-cell type is pretty much unresponsive
to standard chemotherapy, but some newer agents (like rituximab) seem to work a little better.
Survival is still dismal though.
PL is more aggressive
than other chronic
lymphoproliferative disorders

The Complete (but not obsessive) Hematopathology Guide
page 77
www.pathologystudent.com

Large Granul ated Lymphocyte Leukemi a
Large granulated lymphocyte leukemia (LGLL) is a rare chronic lymphoproliferative disorder of T
cells which are large and display cytoplasmic granules. There are two subtypes: T-cell type and
natural killer cell type.

Cl i ni cal Feat ures
LGLL usually occurs in elderly age ranges. The T-cell type of LGLL is an indolent disease, with
symptoms mostly related to severe neutropenia (recurrent infections are common). The NK type, in
contrast, is a very aggressive disease.

Morphol ogy
Blood
The blood smear shows a lot of large lymphocytes with abundant, clear cytoplasm and
cytoplasmic granules. The neutrophil count is decreased sometimes markedly so.
Bone marrow
The bone marrow shows the same population of malignant cells as the blood, as well as the same
decrease in neutrophils.

I mmunophenot ype
The T-cell type of LGLL shows the following immunophenotype:
positive for CD2, CD3, CD8 and CD57
negative for CD4
The NK type of LGL shows the following immunophenotype:
positive for CD2 and CD56
negative for CD3, CD4 and CD8

Treat ment and prognosi s
Patients with the T-cell type respond well to low-dose traditional chemotherapy, and have long-
lasting remissions, with a mean survival of 10 years. The NK type, however, does not respond to
chemotherapy, and has a short survival.



The Complete (but not obsessive) Hematopathology Guide
page 78
www.pathologystudent.com

5. Mul ti pl e Myel oma

Multiple myeloma is a malignant, clonal disorder of plasma cells which originates in the bone
marrow. Three key features are present:
1. A monoclonal proliferation of plasma cells
2. A monoclonal gammopathy
3. Decreased levels of other, normal (polyclonal) immunoglobulins.

Cl i ni cal Features
Myeloma is a relatively common disorder (1% of all malignancies and 10% of all hematologic
malignancies in adults). It occurs in adults (rare <35 years old; highest incidence in 50s).
Signs and symptoms include:
Bone pain (back, extremities) from lytic lesions
Weakness
Recurrent, persistent infections (big problem)
Renal failure (multifactorial; due in large part to toxic effect of light chains on
tubular epithelium)
Hypercalcemia (from bone resorption) can lead to neurologic changes and
contribute to renal disease.

Pathogenesi s
Myeloma originates in a stem cell which decides to differentiate along the B-cell/plasma cell
pathway. Cytokines (especially IL-6) induce differentiation and proliferation of the malignant cells.
Cytokines (released by the plasma cells) also activate osteoclasts, causing bone resorption.

Morphol ogi c Fi ndi ngs
Bl ood
Plasma cells are rarely seen in the blood in myeloma! What you do see is an anemia, usually, and
an unusual finding called rouleaux. In rouleaux, the red cells stack up on each other in little
columns, like stacks of coins, or poker chips, depending on your favorite metaphor.
Bone marrow
The bone marrow shows an increased number of plasma cells (the normal plasma cell number is
<3%; in myeloma, plasma cells usually number 20% or more), usually arranged in big sheets.
Roul eaux

The Complete (but not obsessive) Hematopathology Guide
page 79
www.pathologystudent.com
Plasma cells may look normal (mature type of myeloma), slightly immature (intermediate type),
or barely recognizable as plasma cells (plasmablastic type).
Plasma cells in myeloma sometimes have unusual-looking inclusions, like crystals, Chinese-letter-
like inclusions, Russell bodies (small, reddish cytoplasmic blobs containing immunoglobulin), and
Dutcher bodies (large, single, intranuclear pseudoinclusions that look like holes and contain
immunoglobulin).
Some patients have amyloidosis due to over-production of immunoglobulin light chains. One place
to look for this is in vessel walls. Amyloid looks pink and smudgy on hematoxylin and eosin
staining, but when you stain it with Congo red and look at it under fluorescent light, it looks apple-
green.

Laboratory Fi ndi ngs
Almost all patients with myeloma have a big monoclonal gammopathy in the blood, or urine, or
both. All those monoclonal plasma cells are making the same immunoglobulin, and you can detect
that immunoglobulin using electrophoresis (which separates the serum or urine proteins into
groups based on charge; the monoclonal immunoglobulin shows up as a big spike in the gamma
region). Finding a monoclonal immunoglobulin is important in the diagnosis of myeloma. Its also
helpful in following patients; you can do periodic electrophoreses to make sure that the monoclonal
immunoglobulin isnt creeping up, which would mean disease progression.
The most common heavy chain expressed in myeloma is IgG (60%); next is IgA (20%). Rare cases
express IgD or IgE. IgM myeloma is virtually nonexistent (most cases of plasma cell lesions that
express IgM turn out to be Waldenstrm macroglobulinemia). Almost one-fifth of all cases of
myeloma have plasma cells that secrete only light chains (called Bence-Jones proteins). These light
chains pass through the kidney and out into the urine. Therefore, you need to look at urine as well
as serum ELPs when youre diagnosing myeloma
In myeloma, the total amount of immunoglobulin is increased but most of this is due to the large
monoclonal immunoglobulin spike. The normal (polyclonal) immunoglobulins are actually
decreased!

Treatment and Prognosi s
Current chemotherapy may decrease symptoms, but does not prolong life. Right now, bone
marrow transplantation is the only hope for cure. Hopefully, though, as new drugs are developed,
someone will find one that targets only the malignant plasma cells, and does a better job of killing
them. Prognosis without bone marrow transplantation is poor (median survival = 3 years).
Things to make you look smart
Q. How come you see roul eaux
i n myel oma?
A. Rouleaux occurs in myeloma
because the huge amount of
immunoglobulin floating around
breaks the zeta potential (the normal
repellant force between red cells),
allowing the red cells to stack up on
each other in piles. Other large
molecules, like fibrinogen (an acute
phase reactant that goes up in
certain conditions, like infections
and connective tissue diseases),
can also cause rouleaux. But the
most common cause of rouleaux is
artifact (if you look in the thick part of
a normal blood smear, youll see it; on
a poorly-made, thick smear, youll see
it all over). So when you see rouleaux
on a blood smear, make sure youre
looking in the right area, that the
smear has been made properly, and
that the patient doesnt have some
benign cause for the rouleaux (like a
nasty infection). Then, go ahead and
look for myeloma!
Patients with myeloma have a
big monoclonal Ig spike
in the blood or urine

The Complete (but not obsessive) Hematopathology Guide
page 80
www.pathologystudent.com
6. Lymph Node Di sorders

Before we take a look at lymph node pathology, lets do a quick review of lymph node histology.
Lymph nodes are composed of two main regions: follicles and the interfollicular area. Primary
follicles are round collections of small, mature, unstimulated B cells. You only see these in nodes
that have not been exposed to antigens.
After stimulation (say, from an infection), primary follicles turn into secondary follicles. These guys
are more interesting. They have a germinal center, which is a pale area in the center of the follicle
that contains a lot of very heterogeneous lymphocytes. Some are small and irregularly-shaped;
others are larger and have nucleoli. There are also macrophages that have ingested cell debris
(these are called tingible-body macrophages"). Around the germinal center is the mantle zone
(composed of small, unstimulated lymphocytes). This zone looks darker because the cells are
smaller, with little cytoplasm so they sit closer together and look darker at low power.
The interfollicular area contains T cells. It is a pretty heterogeneous mixture of cells too big ones,
little ones, and everything in between.

Benign (Reactive) Lymph Node Disorders
Benign lymph node disorders are way more common than lymphoma (discussed next). Its
important to be able to tell benign things apart from lymphoma; well look at a few clues in a
minute.

Fol l i cul ar hyperpl asi a
Morphol ogy
The lymph node shows numerous, irregularly-shaped, enlarged secondary follicles containing
benign follicle center cells (a heterogeneous mixture of lymphocytes) and tingible-body
macrophages.
Causes
Follicular hyperplasia is a B-cell response to an immune stimulus of some kind. Sometimes, there
are morphologic clues that point towards a specific cause (such as rheumatoid arthritis or HIV).
Most of the time, though, we cant tell the precise cause.

The Complete (but not obsessive) Hematopathology Guide
page 81
www.pathologystudent.com

I nterf ol l i cul ar hyperpl asi a
Morphol ogy
The lymph node shows an expansion of the areas between follicles. If you look closely (and use an
immunohistochemical stain for T cells), youll see a mixture of cells: T cells, macrophages, and
eosinophils. Sometimes, youll see "partial effacement," meaning that the interfollicular area is so
expanded that the overall architecture of the node is hard to see but there are still at least a few
remaining follicles. If the node is totally effaced by some process, thats more indicative of
malignancy.
Causes
Interfollicular hyperplasia represents a T-cell response to an immune stimulus of some kind.
Common stimuli include Epstein-Barr virus (which causes infectious mononucleosis), drugs
(especially dilantin) and vaccines.

Beni gn vs. Mal i gnant
Its easy to confuse benign follicular hyperplasia with lymphoma. They both have a bunch of little
round things! Whats a person to do? Here are some clues.
Beni gn ( f ol l i cul ar hyperpl asi a) Mal i gnant ( f ol l i cul ar l ymphoma)
Architecture preserved Architecture effaced
Follicles vary a lot in size, shape Follicles are of uniform size, shape
Germinal center: Germinal center:
Nice mantle zones No mantle zones
Heterogeneous lymphocytes More monotonous lymphocytes
Tingible-body macrophages No tingible-body macrophages




The Complete (but not obsessive) Hematopathology Guide
page 82
www.pathologystudent.com

Lymphoma
Some big-picture stuff first. Lymphoma is a malignant disorder of lymphocytes that starts in a
lymph node. As opposed to leukemia, which is a malignant disorder of hematopoietic cells
(myeloid or lymphoid) that starts in the bone marrow. In some cases, the difference is semantic
the same disease can present either in the marrow or in lymph nodes, but the cells are the same
morphologically and immunophenotypically, and the prognosis is the same. Those lymphomas and
leukemias have been paired up now, which is nice for us, because that makes for less to
memorize. Weve already mentioned a bunch of these; well review them here as they come up.
Non-Hodgki n vs. Hodgki n l ymphoma
There are two big kinds of lymphoma: non-Hodgkin lymphoma and Hodgkin lymphoma. They are
very different! Non-Hodgkin lymphoma has a bazillion (well, not a bazillion, but a lot) of different
subtypes, and the classification schemes have changed so often that its embarrassing. It starts in
a lymph node somewhere, but then can jump around to another, far-off place in the body without
warning. The prognosis is variable, depending on the subtype but its often not so great.
Hodgkin lymphoma has only a few subtypes, and theyre pretty well-defined. It starts in a lymph
node somewhere, and progresses to contiguous lymph nodes in a pretty predictable fashion. The
prognosis is generally good.
Well discuss non-Hodgkin lymphoma first, and then move on to Hodgkin lymphoma. If you keep
these major points in mind, it will help with the big picture.

The Complete (but not obsessive) Hematopathology Guide
page 83
www.pathologystudent.com

Non-Hodgkin Lymphoma

Cl assi f i cati on( s)
Over the years, there have been so many different pathology-driven classifications that after a
while, clinicians basically threw up their hands and said, whatever. Just tell me how the lymphoma
is going to act in my patient. Is it really bad (high-grade), just sort of bad (intermediate-grade) or not
so bad (low-grade)? And so the Working Formulation was born. Pathologists scoffed (they
preferred to organize the lymphomas by morphology, or immunophenotype, or something more
esoteric), but the classification (not really a classification, but a formulation) took hold. Youll still
hear people using it today, even though were a couple classifications beyond it now, because its
just so straightforward. Here it is:
Worki ng f ormul at i on
Low-Grade Lymphomas
Small lymphocytic lymphoma
Follicular small cleaved cell lymphoma
Follicular mixed small cleaved and large cell lymphoma
I nt ermedi at e-Grade Lymphomas
Follicular large cell lymphoma
Diffuse small cleaved cell lymphoma
Diffuse mixed small and large cell lymphoma
Diffuse large cell lymphoma
Hi gh-Grade Lymphomas
Large cell immunoblastic lymphoma
Lymphoblastic lymphoma
Small non-cleaved cell (Burkitt) lymphoma

Several years later, the pathologists couldnt stand it anymore. Together with some clinicians, they
devised a new classification, the so-called REAL classification. Which they said stood for revised
European-American lymphoma classification, but which probably really meant, this is the real,
true classification, the classification of all classifications, making all prior classifications null and
stupid-looking. Something like that. You can find it in the reference section if you care (on p. 100).

The Complete (but not obsessive) Hematopathology Guide
page 84
www.pathologystudent.com

So, to make a long story short, were now using a classification scheme developed by the World
Health Organization. Its divided into B and T-cell lesions, and within those divisions, there are
precursor neoplasms (basically, those that involve lymphoblasts) and mature neoplasms (ones that
involve cells that are beyond the blast stage). It also has a separate section for Hodgkin lymphoma.
Here it is (note that there are several types of lymphoma that we are not covering; thats because
they are so rare that you will not see them on boards, and probably never in a patient either):

WHO cl assi f i cat i on




















B-cel l neopl asms
Precursor B-cell neoplasm:
Precursor B-lymphoblastic leukemia/lymphoma
Mature (peripheral) B-cell neoplasms:
B-cell CLL/SLL
B-cell prolymphocytic leukemia
Lymphoplasmacytic lymphoma
Splenic marginal zone B-cell lymphoma
(+/- villous lymphocytes)
Nodal marginal zone lymphoma
(+/- monocytoid B cells)
Extranodal marginal zone B-cell lymphoma of
mucosa-associated lymphoid tissue (MALT) type
Hairy cell leukemia
Plasma cell myeloma/plasmacytoma
Follicular lymphoma, follicle center
Mantle cell lymphoma
Diffuse large B-cell lymphoma
Mediastinal large B-cell lymphoma
Intravascular large B-cell lymphoma
Primary effusion lymphoma
Burkitt lymphoma/Burkitt cell leukemia

T- and NK-cel l neopl asms
Precursor T-cell neoplasm
Precursor T-lymphoblastic leukemia/lymphoma
Mature (peripheral) T-cell and NK-cell neoplasms:
T-cell prolymphocytic leukemia
T-cell granular lymphocytic leukemia
Aggressive NK cell leukemia
Adult T-cell leukemia/lymphoma
Extranodal NK/T-cell lymphoma, nasal type
Enteropathy-type T-cell lymphoma
Hepatosplenic gamma-delta T-cell lymphoma
Subcutaneous panniculitis-like T-cell lymphoma
Mycosis fungoides/Szary syndrome
Primary cutaneous anaplastic large cell lymphoma T/null cell
Peripheral T-cell lymphoma, unspecified
Angioimmunoblastic T-cell lymphoma
Primary systemic anaplastic large cell lymphoma, T/null cell

Hodgki n Di sease
Lymphocyte predominance Hodgkin Disease
Classical Hodgkin Disease
Nodular sclerosis
Mixed cellularity
Lymphocyte depletion
Lymphocyte rich


The Complete (but not obsessive) Hematopathology Guide
page 85
www.pathologystudent.com

Overvi ew of non-Hodgki n l ymphoma
Non-Hodgkin lymphoma (NHL) is a malignant disorder of lymphocytes that originates in a single
lymph node or in lymphoid tissue. It usually spreads to other, non-contiguous lymph nodes and
organs (spleen, liver, bone marrow, others). It may spill over into the blood too. Fortunately, it is a
pretty rare disease (13 cases/100,000 people).
I mmunophenot ype
85% of cases are of B-cell immunophenotype (they express CD19, 20, 22, and surface
immunoglobulin and often have immunoglobulin gene rearrangements). 15% of cases are of T-cell
immunophenotype (they express CD3, 4, 7, 8 and often have T-cell receptor gene
rearrangements).
Low-grade vs. hi gh-grade

















For our discussion, well group the lymphomas into low-grade lymphomas (small lymphocytic
lymphoma, marginal zone lymphoma, mantle cell lymphoma, follicular lymphoma, and mycosis
fungoides/Szary syndrome) and high-grade lymphomas (diffuse large B-cell lymphoma,
lymphoblastic lymphoma, Burkitt lymphoma, and adult T-cell leukemia/lymphoma).
Morphol ogy
As youre going through the morphology of the different lymphomas, pay attention to two things:
the pattern (the way the cells are arranged) and the cytology (the way the individual cells look).
Sometimes one or the other is diagnostic like, for example, proliferation centers in small
lymphocytic lymphoma, or the weird cell morphology in Burkitt lymphoma.
Low-grade
Older patients (rarely affects the young)
Indolent, yet therapy doesnt really help
Multiple, painless, enlarged lymph nodes
Extranodal involvement rare (except bone
marrow, which is usually positive)
Small cells with clumped chromatin,
inconspicuous nucleoli
Follicular or diffuse pattern
Non-destructive growth

Hi gh-grade
Patients often older patients (but occurs
in children too)
Aggressive, but therapy sometimes cures
Extranodal involvement common (except
bone marrow, which is usually negative)
Large cells with large nuclei, open
chromatin, prominent nucleoli
Diffuse pattern
Destructive to surrounding tissues

Low-grade lymphomas:
small lymphocytic lymphoma
marginal zone lymphoma
mantle cell lymphoma
follicular lymphoma
mycosis fungoides/Szary syndrome
High-grade lymphomas:
diffuse large B-cell lymphoma
lymphoblastic lymphoma
Burkitt lymphoma
adult T-cell leukemia/lymphoma

The Complete (but not obsessive) Hematopathology Guide
page 86
www.pathologystudent.com
Smal l l ymphocyti c l ymphoma
SLL is identical to chronic lymphocytic leukemia (CLL) morphologically, immunophenotypically, and
clinically. They were initially described differently (CLL was described in the blood, and SLL was
described in nodes) but now that we know how to do flow and all kinds of smarty-pants tests, its
obvious that theyre the same disease.
Cl i ni cal f eat ures
SLL is a disease that has an indolent but relentless clinical course (the mean survival is 10 years).
Some cases evolve into large-cell lymphoma. This is called Richter's transformation, and it is an
ominous sign.
Morphol ogy
Pattern: Diffuse (small lymphocytes fill up the whole lymph node), with proliferation centers.
Proliferation centers are pale, cloud-like areas visible at low power. They contain
prolymphocytes and paraimmunoblasts, which are larger cells, with fine chromatin and
prominent nucleoli. When you see these in a background of small, mature lymphocytes,
its pretty much a clincher for SLL.
Cytology: Small, round lymphocytes with clumped chromatin, scant cytoplasm. They look just like
benign, mature lymphocytes!

I mmunophenot ype
CLL/SLL is a B-cell leukemia/lymphoma that aberrantly expresses CD5 (see p. 73 for more detail).

Things to make you look smart
Q. What s i n t hose l i ght areas at l ow power? What s i n t he dark areas?
A. When youre looking at a lymph node at low power, sometimes youll see dark areas next to
lighter, paler areas. Generally, when you see a dark area, it means that the cells are small, with
condensed (dark) chromatin and little cytoplasm (so the dark nuclei are closely apposed, giving a
dark low-power appearance). Lighter areas are usually composed of larger cells, with more open
(clear or light) chromatin and abundant cytoplasm (this makes the nuclei spread apart more, adding
to the pale low-power appearance).
For a good example of this principle, check out a regular old secondary follicle. The germinal center
is pale at low power, and at high power, you can see a mixture of all kinds of cells, most of which
are big, with open chromatin and lots of cytoplasm. The mantle zone, in contrast, is dark at low
power, and at high power youll see a bunch of small lymphocytes with condensed chromatin and
very little cytoplasm. Proliferation centers work on the same principle: low power = pale, high
power = cells with open chromatin and abundant cytoplasm.
Small lymphocytic lymphoma
has proliferation centers
at low power.

The Complete (but not obsessive) Hematopathology Guide
page 87
www.pathologystudent.com

Margi nal zone l ymphoma
The term "marginal zone lymphoma" encompasses a bunch of B-lineage lymphomas including
MALT (mucosa-associated lymphoid tissue) lymphoma, monocytoid B-cell lymphoma, and splenic
lymphoma with villous lymphocytes. Well talk about MALT since its the one youll probably hear
about the most.

Cl i ni cal f eat ures of MALT l ymphoma
MALT lymphoma most commonly occurs in stomach (but can occur anywhere there is mucosa-
associated lymphoid tissue). It usually stays localized and acts like a low-grade lymphoma. Heres
something weird: in the stomach, MALT lymphoma is strongly associated with Helicobacter pylori
infection and it may even go away if you get rid of the H. pylori! Weird but true.
Morphol ogy
Pattern: Early on, this lymphoma tries to be just like the marginal zones in splenic follicles (which
are pale rings around the outside of the follicle, adjacent to the mantle zone). So youll
see a bunch of follicles with big marginal zones (which you dont normally see in lymph
nodes). Later, it becomes diffuse.
Cytology: Variety of cell types present (from small lymphocytes to larger ones with lots of
cytoplasm).

Mantl e cel l l ymphoma
Mantle cell lymphoma is a B-lineage lymphoma named after its pattern. You should try to
remember the translocation!
Cl i ni cal f eat ures
This lymphoma, though classified as a low-grade lymphoma, is on the aggressive end of low-
grade. The mean survival is 4 years.
Morphol ogy
Pattern: Follicular, with expanded mantle zones (early) or diffuse (later on).
Cytology: Small, angulated lymphocytes (just like those in regular old mantle zones).
Transl ocat i on
Most cases of mantle cell lymphoma have a t(11;14) translocation which involves the bcl-1 gene
(on chromosome 11) and immunoglobulin heavy chain gene (on chromosome 14).
High-grade lymphomas:
diffuse large B-cell lymphoma
lymphoblastic lymphoma
Burkitt lymphoma
adult T-cell leukemia/lymphoma
Mantle cell lymphoma has a t(11;14).
Small lymphocytic lymphoma
has proliferation centers
at low power.

The Complete (but not obsessive) Hematopathology Guide
page 88
www.pathologystudent.com


Fol l i cul ar l ymphoma
This is the one of the most common types of non-Hodgkin lymphoma. Check out the grading and
the translocation!
Morphol ogy
Pattern: Follicular (can later become diffuse).
Cytology: Small cleaved cells and/or large cells.
Follicular lymphoma is graded based on the number of large cells present within the
follicles. Grade 1 has mostly small cells, grade 2 has similar numbers of small and large
cells, and grade 3 has mostly large cells. The higher the grade, the worse the prognosis.
When grade 1 follicular lymphoma gets into the blood, you can see cleaved cells floating
around. They have a lovely name: butt cells.
I mmunophenot ype
B-cell.
Transl ocat i on
Most cases of follicular lymphoma have a t(14;18) translocation, which involves the bcl-2 gene (an
anti-apoptosis gene on chromosome 18) and the immunoglobulin heavy chain gene (on
chromosome 14).
Cl i ni cal f eat ures
This is a low-grade lymphoma, with a fairly long survival. If disease is confined to one node, or to
two nodes on the same side of the diaphragm, 5-year survival is 90%. If lymph nodes on both
sides of the diaphragm are involved, or if there is widespread extranodal involvement, 5-year
survival drops to 40%.

Follicular lymphoma
has a t(14;18).
Fol l i cul ar l ymphoma: bl ood

The Complete (but not obsessive) Hematopathology Guide
page 89
www.pathologystudent.com
Mycosi s f ungoi des/Szary syndrome
Heres a weird lymphoma. First of all, its a T-cell lymphoma. Secondly, the cells look really weird.
Thirdly, there is a very specific clinical picture. Its not all that common in real life, but it makes a
disproportionately frequent appearance on exams, probably because its so easy to write
questions about all the unusual characteristics.
Cl i ni cal f eat ures
Mycosis fungoides (toadstools! toadstools!) was initially described as a lymphoma (though now
its known to be the same as Szary syndrome (see below). Patients get skin lesions that start as
flat, red, inflamed-looking patches (which can be easily mistaken for psoriasis or eczema), then
progress to plaque-like (slightly raised) lesions, and wind up as nodules (big mushroomy bumps).
Szary syndrome was initially described as a leukemia, with tons of these weird malignant T cells in
the blood. The overall prognosis is good.
Morphol ogy
Pattern: Lymph nodes are not often involved. When they are, the pattern may be diffuse, or
there may be partial involvement in the interfollicular areas.
Skin lesions are common, and often consist of little collections of lymphoma cells in the
epidermis called Pautrier microabscesses. Which is a misleading name, because they
are not abscesses at all they are collections of tumor cells! Oh well.
Cytology: The characteristic cell in mycosis fungoides/Szary syndrome is a medium to large cell
with an unusual convoluted, cerebriform nucleus. The contours really do look like the
gyri and sulci of the brain very unique. This is one of those words in pathology that
you should remember and associate with this particular disease. People just dont use
this word when describing other disorders. You can see these cells in the blood and/or
in the skin.








Things to make you look smart
Q. What f eat ures ( besi des i mmunophenot ype) di st i ngui sh T-cel l l ymphomas f rom B-
cel l l ymphomas?
A. T-cell lymphomas have a few unique features. First, they love to go to the skin. Both mycosis
fungoides/Szary syndrome and adult T-cell leukemia/lymphoma have skin lesions. Also, they tend to
have more weird-looking nuclear contours. In mycosis fungoides/Szary syndrome the nuclei are
cerebriform; in adult T-cell leukemia/lymphoma, they are flowery and showy. B-cell lymphomas can
involve the skin and have weird nuclei but not nearly as commonly as T-cell lymphomas.
Mycosis fungoides:
T-cell immunophenotype, skin lesions and
cerebriform lymphocytes

The Complete (but not obsessive) Hematopathology Guide
page 90
www.pathologystudent.com



Di f f use l arge B-cel l l ymphoma
Diffuse large cell lymphoma is one of the most common types of non-Hodgkin lymphoma. Notable
features are its poor prognosis and distinctive cytology.
Cl i ni cal f eat ures
This is an aggressive lymphoma that tends to present early with extranodal (e.g., spleen)
involvement. It is a rapidly growing lymphoma, and the prognosis is correspondingly poor.
Morphol ogy
Pattern: Diffuse (obviously). Often, this lymphoma evolves from a grade 3 (large cell) follicular
lymphoma so you may see a few residual malignant follicles here and there.
Cytology: The cells are large (duh) and they generally have prominent nucleoli. There are a bunch
of different subtypes (some cases are composed of cells that look like immunoblasts
big, oafish cells with a central nucleolus and others are composed of just big nasty-
looking cells. It doesnt matter. The prognosis is the same for all subtypes: bad.

Lymphobl asti c l ymphoma
Heres another couple of leukemia/lymphoma pairings. Lymphoblastic lymphoma can be either T
or B-lineage. T-lineage lymphoblastic lymphoma is the same as T-cell ALL (theyre lumped
together now as precursor T-cell leukemia/lymphoma); B-lineage lymphoblastic lymphoma is the
same as precursor B-cell ALL (merged name: precursor B-cell leukemia/lymphoma). Go back to
page 60 and check out the subtypes of ALL if you need a refresher.
Morphol ogy
Pattern: Diffuse.
Cytology: Medium-sized cells with fine chromatin, scant cytoplasm, high mitotic rate (if you smear
a little bit out on a slide, they look just like blasts! Because they are!).
Cl i ni cal f eat ures
Most patients are younger (<20). T-cell lymphoblastic lymphoma often presents in a teenage male
with a mediastinal mass.


Large-cell lymphoma
Lymphoma cells spreading into fat

Large-cell lymphoma
Large cells with fine chromatin


The Complete (but not obsessive) Hematopathology Guide
page 91
www.pathologystudent.com

Burki tt Lymphoma
Burkitt lymphoma is identical to B-cell acute lymphoblastic leukemia (ALL) morphologically,
immunophenotypically and clinically. Nice new combined name: Burkitt leukemia/lymphoma. You
might still hear old-timers use the old name for Burkitt lymphoma: small non-cleaved cell
lymphoma.
Morphol ogy
Pattern: Diffuse. The lymph node is totally replaced by these weird lymphoblasts (with big nuclei
and deep blue cytoplasm). The cells have such a rapid turnover rate that there are
conspicuous tingible body macrophages scattered all over. This unique histologic
appearance is often described as a starry sky appearance (the tumor cells are the
sky, and the tingible body macrophages are the stars). You should burn this phrase into
your brain. Its another one of those words/phrases that is used only in one specific
context.
Cytology: Same as we discussed for B-cell ALL: medium-sized cells with prominent nucleoli,
scant cytoplasm, very high mitotic rate.
Transl ocat i on
Go back to page 61 if you need a quick refresher on the t(8;14).
Cl i ni cal Feat ures
This lymphoma is more common in children and young adults. Extranodal presentation is the rule.
In African cases, the maxilla or mandible is commonly affected; in the US, patients usually present
with an abdominal mass.











Things to make you look smart
Q. Wai t , di dn t we t al k about t i ngi bl e body macrophages as bei ng present i n beni gn
germi nal cent ers?
A. Why yes, we did! They are present in both benign germinal centers and Burkitt lymphoma
theyre just innocent bystanders trying to eat up debris from rapidly turning over cells. So how do you
tell the difference between the benign thing with tingible body macrophages and the malignant thing
with tingible body macrophages? Easy: the benign thing has a bunch of follicles, with germinal
centers (with tingible body macrophages inside) and mantle zones. The malignant thing (Burkitt
lymphoma) has a bunch of nasty cells in a diffuse pattern (no follicles) with scattered tingible body
macrophages.
Burkitt lymphoma has a
starry-sky pattern
in lymph nodes.
Burki t t l ymphoma

The Complete (but not obsessive) Hematopathology Guide
page 92
www.pathologystudent.com

Adul t T-Cel l Leukemi a/Lymphoma
This is another lymphoma that is relatively uncommon in real life, but pretty frequently hit on exams,
probably because its a T-cell lymphoma with weird-looking cells and some distinctive clinical
features.
Morphol ogy
Pattern: Diffuse.
Cytology: The cells in this lymphoma are mostly big lymphocytes with irregularly-contoured,
flowery nuclei. There are also usually a few smaller, less showy lymphocytes.
Cl i ni cal f eat ures
This leukemia/lymphoma is endemic in Japan and the Caribbean basin, but now is appearing in
the US too (especially in southeastern states). Its associated with HTLV-1 infection. Patients tend
to present with skin lesions, lymphadenopathy, hepatosplenomegaly, and hypercalcemia. This is a
very aggressive disease with a mean survival of 8 months.





Adult T-cell leukemia/lymphoma:
skin lesions, hypercalcemia,
HTLV-1, and flowery lymphocytes

The Complete (but not obsessive) Hematopathology Guide
page 93
www.pathologystudent.com


Hodgkin Disease
Hodgkin disease (or Hodgkin lymphoma) is a malignant disorder of lymphoid cells that originates in
a single lymph node and spreads in a more or less predictable fashion to contiguous lymph nodes.
Untreated, it eventually spreads to distant lymph nodes and organs (spleen, liver, and sometimes
bone marrow). It is only rarely seen in the blood. It is rare overall (3 cases per 100,000 people), but
it is one of most common malignancies in young adults. It has a bimodal distribution, with one
peak at age 20-30 years, and another peak in the 50s.

The Reed-Sternberg cel l
The characteristic cell in Hodgkin disease is a bizarre cell called the Reed-Sternberg (R-S) cell. Its
strange, because when you look at a lymph node involved by Hodgkin disease, you usually have to
look around a bit to find R-S cells! They are sitting singly in a sea of benign background cells (like
eosinophils, plasma cells, and histiocytes). The morphology of the R-S cell is very unusual. Its a
very very large cell, and it has two or more nuclei with surrounding, clear halos, big nucleoli (as
big as the adjacent normal lymphocytes!), and abundant cytoplasm. For a long time, nobody knew
what the heck kind of cell it was (hence the name Hodgkin disease instead of Hodgkin
lymphoma). Even now, its exact origin is unknown, but people pretty much agree that it is a
lymphoid cell of some type.
The R-S cell is seen in varying frequencies in the different subtypes of Hodgkin disease. Some
subtypes have classic R-S cells in relative abundance; others have variants of the R-S cell. The five
subtypes, discussed below, are:
1. Nodular lymphocyte predominance Hodgkin disease
2. Classical Hodgkin disease
Nodular sclerosis Hodgkin disease
Mixed cellularity Hodgkin disease
Lymphocyte rich Hodgkin disease
Lymphocyte depleted Hodgkin disease
Reed-Sternberg cells
or variants
are present in all cases
of Hodgkin disease.

The Complete (but not obsessive) Hematopathology Guide
page 94
www.pathologystudent.com

Nodul ar Lymphocyte Predomi nance Hodgki n Di sease
This type of Hodgkin disease was recently added. Its really a B-cell neoplasm, but its so closely
related to Hodgkin disease that it is now included in the classification (but it gets its own category).
Cl i ni cal Feat ures
This type of Hodgkin disease tends to present in young males. Disease is often limited to cervical
nodes. Prognosis is good, because the disease is usually early-stage, and patients usually respond
well to therapy.
Morphol ogy
Typical Reed-Sternberg cells are uncommon. Instead, there are variants called popcorn cells with
delicate, multilobated, puffy nuclei.

Nodul ar Scl erosi s Hodgki n Di sease
This is the most common of the classical types of Hodgkin disease (around 60% of cases).
Cl i ni cal Feat ures
Many patients present with limited disease; prognosis is excellent.
Morphol ogy
This type of Hodgkin disease has a special R-S variant called the lacunar cell (a cell with single,
hyperlobated nucleus and abundant, pale-staining cytoplasm). Its called a lacunar cell because
formalin fixation makes the cytoplasm retract, and the cell appears to be sitting in a little space, or
lacuna. Classic R-S cells are rare in this type. Collagen bands divide lymph node tissue into
nodules (hence, "nodular sclerosis").

Mi xed Cel l ul ari ty Hodgki n's Di sease
This is the second most common classical subtype (around 20% of cases).
Cl i ni cal Feat ures
Many patients present with disseminated disease and systemic symptoms.
Morphol ogy
Classic R-S cells are plentiful in this Hodgkin disease variant. The background has abundant
eosinophils, plasma cells, and histiocytes. Its sort of a wastebasket category with lots of different
appearances.

The Complete (but not obsessive) Hematopathology Guide
page 95
www.pathologystudent.com

Lymphocyte Ri ch Hodgki n's Di sease
This is the third most common classical subtype (around 10% of cases).
Cl i ni cal Feat ures
Most patients present with limited disease; prognosis is excellent.
Morphol ogy
This type is characterized by the popcorn cell also present in nodular lymphocyte predominance
Hodgkin disease. Classic R-S cells are hard to find, but present. The background is composed
largely of mature lymphocytes with variable numbers of histiocytes. Eosinophils, neutrophils, and
plasma cells are scanty or absent.

Lymphocyte Depl eti on Hodgki n's Di sease
This is the least common classical subtype (around 5% of cases).
Cl i ni cal f eat ures
Patients are often older, and many present with disseminated involvement and systemic
symptoms. Prognosis is poor.
Morphol ogy
Classic R-S cells and R-S variants are numerous. The background contains either collagen or
reticulin fibers; lymphocytes are rare.

Therapy and Prognosi s
Treatment of Hodgkin disease usually entails surgical removal of the affected lymph node(s) (if
disease is limited), followed by chemotherapy and/or radiation. The prognosis in Hodgkin disease
overall is relatively good. The biggest prognostic factor is stage. Morphologic subtype is not
independently important in predicting prognosis! Five-year survival for localized disease is about
90%; for disseminated disease it is about 60%.
The relatively long survival in Hodgkin disease means patients have time to develop second
malignancies (most commonly acute myeloid leukemia) related to chemotherapy and radiation.
These are very hard to cure.

The most important prognostic factor
in Hodgkin disease is stage.

The Complete (but not obsessive) Hematopathology Guide
page 96
www.pathologystudent.com
Reference Section

Morphol ogy
Bl ast s
How to spot a blast, in general:
High nuclear/cytoplasmic ratio (big nucleus, scant cytoplasm)
Fine chromatin (silk-stocking-like)
Nucleoli
Many times it is impossible to tell whether a blast is lymphoid or myeloid without doing special
stains or markers (see later).
Neut rophi l s and precursors
Myeloblast. Large nucleus with fine chromatin and nucleoli. Thin rim of basophilic cytoplasm. Very
few fine, azurophilic, cytoplasmic granules (or none at all).
Promyelocyte. Biggest cell in neutrophilic series. Prominent nucleoli. Lots of coarse, primary
(azurophilic) granules in cytoplasm and overlying nucleus.
Myelocyte. Coarser chromatin; no nucleoli. Beginning of secondary, or specific (fawn-colored),
granulation.
Metamyelocyte . Indented (kidney-bean-shaped) nucleus.
Neutrophil. Nucleus has multiple lobes (segments) connected by thin strands of nuclear material.
Monocyt es and precursors
Monoblast. Large blast with oval-shaped nucleus.
Promonocyte. Delicate, "tissue-paper" folds and creases in nucleus.
Monocyte. Large cell with grayish ("dishwater") cytoplasm, "raked" chromatin.
Lymphocyt es and precursors
Prolymphocyte. Not normally seen! Large cell with coarse chromatin AND prominent nucleoli
(unusual!)
Lymphoblast. Not normally seen! In ALL, appears as either a regular old blast, or as a large cell
with deep blue cytoplasm and sharply punched out vacuoles.
Lymphocyte. Clumped and smudged chromatin.
Plasma cell . Eccentric nucleus with "clock-face" chromatin; pale, perinuclear hof.

The Complete (but not obsessive) Hematopathology Guide
page 97
www.pathologystudent.com


Markers
T-cel l
Marker Expressed by...
CD2 all T cells
CD3 all peripheral (post-thymic, mature) T cells
CD4 helper/inducer T cells
CD5 all T cells (and small subset of B cells)
CD8 cytotoxic/suppressor T cells

B-cel l
Marker Expressed by...
CD10 developing B cells
CD19 developing and mature B cells
CD20 developing and mature B cells

Granul ocyt e/Monocyt e
Marker Expressed by...
CD13 granulocytes and monocytes
CD14 monocytes
CD15 granulocytes
CD33 myeloid precursors and monocytes

Megakaryocyt e
Marker Expressed by...
CD41 megakaryocytes and platelets
CD61 megakaryocytes and platelets

Pan-l eukocyt e
Marker Expressed by...
CD45 all leukocytes


The Complete (but not obsessive) Hematopathology Guide
page 98
www.pathologystudent.com


Cyt ochemi cal st ai ns
Chl oracet at e est erase ( CAE)
Stains the big eosinophil granules in cases of AML with inv(16) (AML-M4Eo).
Also stains normal neutrophils and mast cells.
Myel operoxi dase ( MPO)
Stains neutrophils and eosinophils (more intense in more mature cells) and occasionally weakly
stains monocytic cells.
Non-speci f i c est erase ( NSE)
Stains monocytic cells and occasionally stains megakaryocytic cells.
Sudan bl ack B ( SBB)
Same staining pattern as MPO. Less commonly used.
Tart rat e-resi st ant aci d phosphat ase ( TRAP)
Stains the hairy cells in hairy cell leukemia. Many other cells (neutrophils, monocytes, platelets)
stain positively with acid phosphatase, but the stain washes away when treated with tartrate.

Cytogeneti c abnormal i ti es
Disorder Chromosomal abnormality Comments
CML t(9;22) Philadelphia chromosome (Ph)
CLL trisomy 12 Poor prognosis
AML-M2 t(8;21) Better prognosis
AML-M3 t(15;17) Good prognosis
AML-M4 inv(16) "M4Eo" - better prognosis
AML-M4 and M5 11q23 abnormalities Poor prognosis
Precursor-B ALL t(9;22) Ph (different breakpoint than in CML)
B-cell ALL t(8;14) c-myc and Ig heavy chain gene


The Complete (but not obsessive) Hematopathology Guide
page 99
www.pathologystudent.com

Lymphoma Classifications
Rappaport Cl assi f i cati on
This is the oldest classification scheme. It was created before lymphocytes were divided into B and
T cells. Some older pathologists and textbooks still use these terms. Pretty simple: three
categories.
Wel l -di f f erent i at ed l ymphoma ( WDL) : what we now call small lymphocytic lymphoma
Poorl y-di f f erent i at ed l ymphoma ( PDL) : what we now call follicular lymphoma, grade 1
Hi st i ocyt i c l ymphoma: What we now call large-cell lymphoma

Worki ng Formul ati on
Put together by a bunch of hematopathologists in 1982, in a feeble attempt to stave off angry
(rightly so!) clinicians. There were all kinds lymphoma classifications around by now (Lukes and
Collins, Kiel, others) and the clinicians didnt care whether the cells were small and round or
large and pointy; they just wanted to know whether it was a bad lymphoma or a not-so-bad
lymphoma. So the hematopathologists lumped all the known lymphomas into three groups: low-
grade (not-so-bad), intermediate grade (sort-of-bad), and high-grade (bad) lymphomas.
Low-Grade Lymphomas
Small lymphocytic lymphoma
Follicular small cleaved cell lymphoma
Follicular mixed small cleaved and large cell lymphoma
I nt ermedi at e-Grade Lymphomas
Follicular large cell lymphoma
Diffuse small cleaved cell lymphoma
Diffuse mixed small and large cell lymphoma
Diffuse large cell lymphoma
Hi gh-Grade Lymphomas
Large cell immunoblastic lymphoma
Lymphoblastic lymphoma
Small non-cleaved cell (Burkitt) lymphoma



The Complete (but not obsessive) Hematopathology Guide
page 100
www.pathologystudent.com

REAL ( Revi sed European-Ameri can) Cl assi f i cati on of Lymphoi d Neopl asms
Put together by another, less frightened group of hematopathologists in 1994. REAL may be a
not-so-subtle way of saying, this is the ultimate lymphoma classification to end all classifications,
or it maybe it's a nod to the old Kiel classification.




























B-cel l neopl asms
Precursor B-cell neoplasm:
Precursor B-lymphoblastic leukemia/lymphoma
Peripheral B-cell neoplasms:
1. B-cell CLL/SLL
2. Lymphoplasmacytic lymphoma
3. Mantle cell lymphoma
4. Follicle center lymphoma
a. grade I (small cell)
b. grade II (mixed)
c. grade III (large cell)
5. Marginal zone lymphoma
6. Splenic marginal zone lymphoma
7. Hairy cell leukemia
8. Plasma cell myeloma (multiple myeloma)
9. Diffuse large B cell lymphoma
10. Burkitt lymphoma


T- and NK-cel l neopl asms
Precursor T-cell neoplasm
Precursor T-lymphoblastic leukemia/lymphoma
Mature (peripheral) T-cell and NK-cell neoplasms:
1. T-cell CLL
2. Large granular lymphocyte leukemia
3. Mycosis fungoides/ Szary syndrome
4. Peripheral T-cell lymphoma
5. Adult T-cell leukemia/lymphoma
6. Anaplastic large cell lymphoma
Hodgki n Di sease
Lymphocyte predominance Hodgkin Disease
Classical Hodgkin Disease
a. Nodular sclerosis
b. Mixed cellularity
c. Lymphocyte depletion
d. Lymphocyte-rich classical Hodgkins disease


The Complete (but not obsessive) Hematopathology Guide
page 101
www.pathologystudent.com
WHO ( Worl d Heal th Organi zati on) Cl assi f i cati on
This is the latest and greatest classification (it was ratified in 2001). Its only slightly different from
the REAL classification just a few minor additions. This is the classification to which you should
refer in the future when you get a patient who has lymphoma or leukemia.
B-cel l neopl asms
Precursor B-cell neoplasm:
Precursor B-lymphoblastic leukemia/lymphoma
Mature (peripheral) B-cell neoplasms:
B-cell CLL/SLL
B-cell prolymphocytic leukemia
Lymphoplasmacytic lymphoma
Splenic marginal zone B-cell lymphoma
(+/- villous lymphocytes)
Nodal marginal zone lymphoma
(+/- monocytoid B cells)
Extranodal marginal zone B-cell lymphoma of
mucosa-associated lymphoid tissue (MALT) type
Hairy cell leukemia
Plasma cell myeloma/plasmacytoma
Follicular lymphoma, follicle center
Mantle cell lymphoma
Diffuse large B-cell lymphoma
Mediastinal large B-cell lymphoma
Intravascular large B-cell lymphoma
Primary effusion lymphoma
Burkitt lymphoma/Burkitt cell leukemia

T- and NK-cel l neopl asms
Precursor T-cell neoplasm
Precursor T-lymphoblastic leukemia/lymphoma
Mature (peripheral) T-cell and NK-cell neoplasms:
T-cell prolymphocytic leukemia
T-cell granular lymphocytic leukemia
Aggressive NK cell leukemia
Adult T-cell leukemia/lymphoma
Extranodal NK/T-cell lymphoma, nasal type
Enteropathy-type T-cell lymphoma
Hepatosplenic gamma-delta T-cell lymphoma
Subcutaneous panniculitis-like T-cell lymphoma
Mycosis fungoides/Szary syndrome
Primary cutaneous anaplastic large cell lymphoma T/null cell
Peripheral T-cell lymphoma, unspecified
Angioimmunoblastic T-cell lymphoma
Primary systemic anaplastic large cell lymphoma, T/null cell

Hodgki n Di sease
Lymphocyte predominance Hodgkin Disease
Classical Hodgkin Disease
Nodular sclerosis
Mixed cellularity
Lymphocyte depletion
Lymphocyte rich


The Complete (but not obsessive) Hematopathology Guide
page 102
www.pathologystudent.com
Study Questions

I ntroducti on

1. Whlch of Lhe followlng lndlces Lells you wheLher Lhe red cells are normochromlc or hypochromlc?
A. 8ed blood cell counL (88C)
8. Mean cell volume (MCv)
C. Pemoglobln
u. PemaLocrlL
L. Mean cell hemoglobln concenLraLlon (MCPC)

2. Whlch of Lhe followlng lndlces Lells you wheLher a paLlenL's red cells are mlcrocyLlc, normocyLlc, or macrocyLlc?
A. Mean cell hemoglobln concenLraLlon (MCPC)
8. 8ed blood cell counL (88C)
C. PemaLocrlL
u. Pemoglobln
L. Mean cell volume (MCv)

3. AnlsocyLosls refers Lo:
A. rollferaLlon of red cells
8. Absence of red cells
C. vlrLual absence of hemoglobln ln red cells
u. varlaLlon ln red cell slze
L. varlaLlon ln red cell shape



The Complete (but not obsessive) Hematopathology Guide
page 103
www.pathologystudent.com
Anemi a

1. An 18-year old, prevlously healLhy male ls rushed Lo Lhe emergency room afLer a farmlng accldenL ln
whlch he suffered masslve blood loss. A lab Lech Lakes a sample of blood lmmedlaLely upon Lhe paLlenL's
arrlval and rushes lL back Lo Lhe laboraLory. Whlch of Lhe followlng ls Lrue?
A. 1he hemoglobln wlll be normal
8. 1he MCv wlll be markedly lncreased
C. 1he 88C wlll be markedly decreased
u. 1he blood smear wlll show an lncrease ln reLlculocyLes
L. 1he red cells wlll be hypochromlc

2. Whlch Lype of anemla lnvolves Lhe aLLachmenL of lgC Lo red cells, whlch Lhen geL eaLen up by
macrophages ln Lhe spleen?
A. Anemla of chronlc dlsease
8. Warm auLolmmune hemolyLlc anemla
C. Cold auLolmmune hemolyLlc anemla
u. PeredlLary spherocyLosls
L. Clucose-6-phosphaLe-dehydrogenase deflclency

3. Whlch of Lhe followlng lab resulLs ls conslsLenL wlLh a hemolyLlc process?
A. uecreased blllrubln
8. uecreased LuP
C. uecreased hapLoglobln
u. uecreased reLlculocyLes

4. Whlch Lype of anemla lnvolves Lhe aLLachmenL of lgM and complemenL Lo red cells?
A. Anemla of chronlc dlsease
8. Warm auLolmmune hemolyLlc anemla
C. Cold auLolmmune hemolyLlc anemla
u. PeredlLary spherocyLosls
L. Clucose-6-phosphaLe-dehydrogenase deflclency


The Complete (but not obsessive) Hematopathology Guide
page 104
www.pathologystudent.com
3. 1he red cells ln slckle cell dlsease are more suscepLlble Lo hemolysls because:
A. 1he red cell membrane ls defecLlve
8. 1hey have less hemoglobln
C. 1hey lack a cell-sLablllzlng enzyme
u. 1hey are especlally yummy Lo macrophages
L. 1hey possess an abnormal hemoglobln whlch polymerlzes upon deoxygenaLlon

6. ?our 28-year-old, healLhy slsLer was found Lo have a low hemoglobln when she Lrled Lo donaLe
blood. Per docLor ordered a C8C, whlch showed Lhe followlng:
Pgb 10 g/dL (12-16) 88C 3.3 x 10
12
/L (4.3-6.0)
MCv 73 fL (80-100) W8C 9.3 x 10
9
/L (4-11)
8uW 13 (12 - 13.3) lL 400 x 10
9
/L (130-430)
WhaL ls Lhe mosL llkely cause of your slsLer's anemla?
A. 1halassemla
8. lron-deflclency anemla
C. PeredlLary spherocyLosls
u. AplasLlc anemla
L. MegaloblasLlc anemla

7. lor many years, desplLe your grandmoLher's nagglng, your grandfaLher has had a very poor and
llmlLed dleL. Pe has developed numbness and weakness ln hls exLremlLles. Pe relucLanLly wenL Lo Lhe
docLor, and a blood smear showed a macrocyLlc anemla wlLh hypersegmenLed neuLrophlls. Whlch of Lhe
followlng deflclencles ls responslble for your grandfaLher's sympLoms?
A. lron
8. vlLamln k
C. vlLamln 8
12

u. vlLamln C
L. vlLamln u


The Complete (but not obsessive) Hematopathology Guide
page 105
www.pathologystudent.com
8. Cne of your paLlenLs, a 63-year-old female, has been havlng progresslvely worsenlng faLlgue and
shorLness of breaLh over Lhe pasL few monLhs. She also has had a few eplsodes of pneumonla, one of
whlch landed her ln Lhe hosplLal, and she has dlffuse peLechlae scaLLered over her arms and neck. Per
C8C shows Lhe followlng:
Pgb 6 g/dL (12-16) 88C 2.3 x 10
12
/L (4.3-6.0)
MCv 90 fL (80-100) W8C 1.2 x 10
9
/L (4-11)
8uW 13 (12 - 13.3) lL 20 x 10
9
/L (130-430)
WhaL ls Lhe mosL llkely cause of your paLlenL's anemla?
A. lron-deflclency anemla
8. Anemla of chronlc dlsease
C. MyelodysplasLlc syndrome
u. AplasLlc anemla
L. PeredlLary spherocyLosls

9. WhaL ls ln a Pelnz body?
A. nuclear fragmenLs
8. 8ough endoplasmlc reLlculum
C. lron
u. uenaLured globln chalns
L. keLchup

10. Mallgnancy, obsLeLrlc compllcaLlons, sepsls and Lrauma can all cause whlch of Lhe followlng flndlngs
on a blood smear?
A. SchlsLocyLes
8. Cval macrocyLes
C. Pelnz bodles
u. 8ed blood cell aggluLlnaLlon
L. Slckle cells



The Complete (but not obsessive) Hematopathology Guide
page 106
www.pathologystudent.com
Beni gn l eukocytoses

1. Whlch of Lhe followlng ls Lrue regardlng Lhe lmmunophenoLype of normal perlpheral blood?
A. 8 cells and 1 cells are presenL ln roughly Lhe same proporLlon
8. 8 cells, 1 cells and nk cells are presenL ln roughly Lhe same proporLlon
C. MosL of Lhe lymphocyLes are nk cells
u. MosL of Lhe lymphocyLes are 8 cells
L. MosL of Lhe lymphocyLes are 1 cells

2. Whlch of Lhe followlng blood flndlngs ls seen mosL commonly ln lnfecLlous mononucleosls?
A. MaLure lymphocyLosls
8. 8eacLlve lymphocyLosls
C. MaLure neuLrophllla
u. LefL shlfL
L. LeukoeryLhroblasLoLlc reacLlon

3. ?our paLlenL has an elevaLed neuLrophll counL. 1he blood smear shows loLs of unusual-looklng
neuLrophlls, some have abundanL, dark-purple cyLoplasmlc granules, and oLhers have loLs of vacuoles ln
Lhe cyLoplasm. Whlch of Lhe followlng dlsorders ls Lhe mosL llkely cause of Lhese flndlngs?
A. L. coll sepsls
8. Lupus
C. SLress
u. urug reacLlon
L. Mycosls fungoldes/Sezary syndrome

4. ?our nexL paLlenL has an elevaLed eoslnophll counL. Whlch of Lhe followlng dlsorders ls Lhe mosL llkely
cause?
A. L. coll sepsls
8. Lupus
C. SLress
u. urug reacLlon
L. Chronlc myeloflbrosls


The Complete (but not obsessive) Hematopathology Guide
page 107
www.pathologystudent.com
3. lL's hard Lo belleve, buL your nexL paLlenL has an elevaLed basophll counL. She should be evaluaLed Lo
see lf she has:
A. L. coll sepsls
8. Lupus
C. SLress
u. urug reacLlon
L. Chronlc myelold leukemla


Leukemi a

1. Whlch of Lhe followlng ls Lrue of acuLe leukemla?
A. SympLoms come on slowly, over a perlod of monLhs or years.
8. 1he bone marrow and blood are full of maLure lymphold or myelold cells.
C. lL almosL never occurs ln adulLs.
u. lf a chlld geLs acuLe leukemla, lL ls mosL ofLen of Lhe myelold llneage.
L. lL ls rapldly faLal lf noL LreaLed lmmedlaLely

2. All of Lhe followlng sLaLemenLs abouL leukemla are Lrue "#$"%&:
A. AcuLe leukemlas Lyplcally presenL wlLh sudden-onseL bone marrow fallure.
8. AcuLe leukemlas are composed of blasLs, and chronlc leukemlas are composed of
maLurlng cells.
C. A 1-cell lmmunophenoLype ls a poor prognosLlc lndlcaLor ln acuLe lymphoblasLlc
leukemla.
u. AcuLe lymphoblasLlc leukemla ls mosL common ln adulLs.
L. Chronlc leukemlas have a prognosls on Lhe order of many years.

3. aLlenLs wlLh whlch of Lhe followlng Lypes of leukemla are aL hlgh rlsk for dlssemlnaLed lnLravascular
coagulaLlon (ulC)?
A. Chronlc myelold leukemla
8. Chronlc lymphocyLlc leukemla
C. AcuLe promyelocyLlc leukemla
u. AcuLe monoblasLlc leukemla
L. AcuLe eryLhroblasLlc leukemla

The Complete (but not obsessive) Hematopathology Guide
page 108
www.pathologystudent.com

4. ?ou are LreaLlng a paLlenL who ls undergolng Lherapy for acuLe leukemla. Pe has dlffuse, masslve
glnglval enlargemenL. Whlch of Lhe followlng cell Lypes does hls leukemla mosL llkely lnvolve?
A. neuLrophll serles
8. MonocyLlc serles
C. LryLhrold serles
u. MegakaryocyLlc serles
L. 8asophlllc serles

3. Auer rods may be seen ln whlch of Lhe followlng dlsorders?
A. AcuLe myelold leukemla
8. AcuLe lymphoblasLlc leukemla
C. Chronlc myelold leukemla
u. Chronlc myeloflbrosls
L. Chronlc lymphocyLlc leukemla

6. Whlch of Lhe followlng dlsorders has a 20 cuLoff for dlagnosls?
A. AcuLe myelold leukemla
8. AcuLe lymphoblasLlc leukemla
C. Chronlc myelold leukemla
u. Chronlc lymphocyLlc leukemla
L. olycyLhemla vera

7. Whlch of Lhe followlng Lypes of acuLe leukemla mosL frequenLly lnvolves exLramedullary slLes?
A. AcuLe promyelocyLlc leukemla
8. AcuLe monoblasLlc leukemla
C. AcuLe eryLhroblasLlc leukemla
u. AcuLe megakaryoblasLlc leukemla
L. AcuLe lymphoblasLlc leukemla


The Complete (but not obsessive) Hematopathology Guide
page 109
www.pathologystudent.com
8. Whlch of Lhe followlng ls a good prognosLlc lndlcaLor ln acuLe lymphoblasLlc leukemla?
A. 1-cell lmmunophenoLype
8. Age under 1 year
C. Age over 10 years
u. WhlLe blood cell counL over 10,000
L. Pyperdlploldy

9. lf a chlld geLs leukemla, more ofLen Lhan noL lL ls usually whlch klnd?
A. Chronlc myelold leukemla
8. Chronlc myeloflbrosls
C. Chronlc lymphocyLlc leukemla
u. AcuLe myelold leukemla
L. AcuLe lymphoblasLlc leukemla

10. Whlch of Lhe followlng ls Lrue of chronlc leukemla?
A. lL can be myelold or lymphold
8. SympLoms usually come on suddenly, over a perlod of several days
C. lL ls [usL as common ln chlldren as lL ls ln adulLs
u. 1he bone marrow and blood are full of blasLs
L. lL ls rapldly faLal lf noL LreaLed lmmedlaLely

11. 1he hlladelphla chromosome ls characLerlsLlc of whlch Lype of leukemla?
A. AcuLe myelold leukemla
8. 1-cell acuLe lymphoblasLlc leukemla
C. Chronlc lymphocyLlc leukemla
u. Chronlc myelold leukemla
L. olycyLhemla vera


The Complete (but not obsessive) Hematopathology Guide
page 110
www.pathologystudent.com
12. A healLhy, 38-year-old male ls found Lo have a mlld lymphocyLosls durlng a rouLlne physlcal
examlnaLlon. All Lhe lymphocyLes appear small and maLure, and lmmunophenoLyplng shows LhaL Lhey
express boLh Cu19 and Cu3. WhaL ls Lhe mosL llkely dlagnosls ln Lhls paLlenL?
A. 8enlgn, reacLlve lymphocyLosls
8. Chronlc lymphocyLlc leukemla/small lymphocyLlc lymphoma
C. 8-cell acuLe lymphoblasLlc leukemla
u. 1-cell acuLe lymphoblasLlc leukemla
L. Mycosls fungoldes/ Sezary syndrome

13. A 60-year-old male has blood smear showlng a marked neuLrophlllc leukocyLosls wlLh a lefL shlfL and
basophllla. Pls marrow ls hypercellular and composed predomlnanLly of neuLrophlls and precursors. A
cyLogeneLlc assay of Lhe neuLrophlls shows a LranslocaLlon beLween chromosomes 9 and 22. Whlch of
Lhe followlng sLaLemenLs ls Lrue regardlng hls dlsease?
A. lL has Lwo phases
8. lL ls always rapldly progresslve
C. lL usually occurs ln chlldren
u. lL may be benlgn
L. 1here ls a new chemoLherapy drug called lmaLlnlb (Cleevec) LhaL has lmproved survlval greaLly ln
mosL paLlenLs wlLh Lhls dlsease

14. 1he four chronlc myeloprollferaLlve dlsorders:
A. usually presenL wlLh pancyLopenla
8. May show splenomegaly
C. Cccur [usL as ofLen ln chlldren as Lhey do ln adulLs
u. Are lndlsLlngulshable from each oLher under Lhe mlcroscope
L. Are usually faLal wlLhln a perlod of monLhs

13. A 60-year-old woman presenLs wlLh a huge spleen and faLlgue. Per blood shows numerous Leardrop-
shaped red cells. Per bone marrow ls flbroLlc. WhaL ls Lhe mosL llkely dlagnosls?
A. AcuLe myelold leukemla
8. lnfecLlon
C. Slckle-cell anemla
u. Chronlc myeloflbrosls
L. Adenocarclnoma

The Complete (but not obsessive) Hematopathology Guide
page 111
www.pathologystudent.com

Myel oma

1. All of Lhe followlng are Lrue of mulLlple myeloma "#$"%&:
A. 1here ls a mallgnanL prollferaLlon of plasma cells ln Lhe bone marrow
8. A monoclonal gammopaLhy ls presenL
C. normal lmmunoglobullns are decreased
u. aLlenLs have osLeolyLlc leslons
L. Lven wlLhouL LreaLmenL, Lhe cllnlcal course ls long, wlLh a mean survlval of 20+ years

2. WhaL ls Lhe mosL commonly-expressed heavy chaln ln mulLlple myeloma'
A. lgA
8. lgu
C. lgL
u. lgC
L. lgM

Lymphoma

1. Cne of your paLlenLs had a gasLrlc lymphoma LhaL was successfully LreaLed wlLh anLlbloLlcs. Whlch of
Lhe followlng dld she mosL llkely have?
A. Small lymphocyLlc lymphoma
8. Mucosa-assoclaLed lymphold Llssue (MAL1) lymphoma
C. lolllcular cell lymphoma
u. Mycosls fungoldes/Sezary syndrome
L. LymphoblasLlc lymphoma

2. Whlle examlnlng a 33-year-old male, you noLlce he has a large lymph node ln hls neck. lL ls moblle and
Lender. 1he mosL llkely cause of hls sympLoms ls:
A. lnfecLlon
8. MeLasLaLlc squamous cell carclnoma
C. 8urklLL lymphoma
u. Podgkln lymphoma
L. Chronlc lymphocyLlc leukemla/small lymphocyLlc lymphoma

The Complete (but not obsessive) Hematopathology Guide
page 112
www.pathologystudent.com

3. 1lnglble-body macrophages may be seen ln:
A. 8urklLL lymphoma
8. 8enlgn germlnal cenLers
C. 8oLh
u. nelLher

4. A 68-year-old male has large, scaly, red plaques on hls back and buLLocks. A skln blopsy shows small
abscess-llke spaces ln Lhe dermls, fllled wlLh cerebrlform lymphocyLes. Pls blood conLalns Lhe same cells,
whlch on lmmunophenoLyplng dlsplay 1-cell markers. WhaL ls Lhe mosL llkely dlagnosls?
A. Small lymphocyLlc lymphoma
8. 1-cell acuLe lymphoblasLlc lymphoma
C. ulffuse large cell lymphoma
u. Mycosls fungoldes/Sezary syndrome
L. 8urklLL lymphoma


3. Whlch of Lhe followlng ls Lrue regardlng Podgkln lymphoma?
A. lL generally acLs more aggresslvely Lhan non-Podgkln lymphoma
8. 1he mallgnanL cell ls Lhe 8eed-SLernberg cell
C. lL vlrLually always occurs ln elderly paLlenLs
u. lL ofLen presenLs ln one lymph node, Lhen sklps" Lo a dlsLanL lymph node
L. Second mallgnancles vlrLually never occur ln paLlenLs wlLh Podgkln lymphoma




The Complete (but not obsessive) Hematopathology Guide
page 113
www.pathologystudent.com
Answers to Study Questions


I ntroducti on
1. L
2. L
3. u


Anemi a
1. A
2. 8
3. C
4. C
3. L
6. 8
7. C
8. u
9. u
10. A

Beni gn l eukocytoses
1. L
2. 8
3. A
4. u
3. L

Leukemi a
1. L
2. u
3. C
4. 8
3. A
6. A
7. 8
8. L
9. L
10. A
11. u
12. 8
13. L
14. 8
13. u

Myel oma
1. L
2. u

Lymphoma
1. 8
2. A
3. C
4. u
3. 8

You might also like